Chapter 2

Groups

Wherever groups disclose themselves, or could be introduced, simplicity crystallizes out of complete chaos.

—Eric Temple Bell

The origin of the modern theory of groups lies in the theory of equations. By the beginning of the nineteenth century, mathematicians had developed formulas for finding the roots of any cubic or quartic equation (analogous to the quadratic formula), and the best mathematicians of the day were trying to find such a formula for the quintic. It thus came as a great surprise when, in 1824, Niels Henrik Abel proved that no such formula exists. At about the same time, Evariste Galois showed that any equation of degree n has an associated group of permutations of the roots of the equation (that is, a set of permutations closed under compositions and inverses). He proved that the equation is solvable if and only if this group has a certain property (now called a solvable group). In particular, the fact that the group An of even permutations is not solvable for any n ≥ 5 implies that no formula exists for solving equations of degree n ≥ 5. This spectacular achievement led to modern Galois theory, but Galois' work went unrecognized until after his death at age 20.

Galois worked with groups of permutations. Then, in 1854, Arthur Cayley formulated the abstract group concept. While the study of permutation groups continues to occupy mathematicians, the abstract theory has the advantage that it isolates those properties of groups that do not depend on the underlying permutations and so can be applied more broadly. We pursue the abstract theory in this chapter (and in Chapters 7–9) with permutation groups as one of the most important examples. 71

2.1 Binary Operations

Abstract algebra is primarily concerned with the study of operations analogous to the addition and multiplication of numbers. We define such operations in this section and examine some of their general properties. The addition process for numbers assigns to any ordered pair (a, b) of numbers a new number, their sum, denoted a + b. Similarly, multiplication assigns the product ab to the pair (a, b).

In general, a binary operation * on a set M is a mapping that assigns to each ordered pair (a, b) of elements of M an element a * b of M. In this case M is said to be closed under the binary operation. Binary operations are usually denoted by other symbols (for example, + for numbers) but, for the moment, we use the generic notation a * b.

A binary operation * is called commutative if a * b = b * a for all a, b in M, and * is called associative if a * (b * c) = (a * b) * c for all a, b, c in M. An element e in M is called a unity (or an identity)18 for * if a * e = a = e * a for all a in M. The unity for a binary operation is often denoted by different symbols (for example, 0 and 1 are the identities for addition and multiplication of numbers, respectively).

Theorem 1. If a binary operation has a unity, that unity is unique.

Proof. If e and f are both unities, then f = e * f and e * f = e. So e = f.

A set M is called a monoid if a binary operation is defined on M that is associative and has an unity19. We say that (M, *) is a monoid if the operation * is to be emphasized. If the operation is commutative, we say that M is a commutative monoid.

Example 1. The sets img are all commutative monoids under both addition and multiplication. The additive unity is 0 in all cases (img in img), and the multiplicative unity is 1 (img in img).

Example 2. The set img of all n × n real matrices is a monoid under both matrix addition and matrix multiplication, the unities being 0 and I, respectively. The monoid img is commutative. However, img is not commutative if n ≥ 2 (the proof of associativity is given in Appendix B).

Example 3. If U is a set, let M = {X img XU} denote the set of all subsets of U. Then (M, ∪) and (M, ∩) are both commutative monoids, the unities being ∅ and U, respectively.

Example 4. Sn is a monoid with unity ε, and it is noncommutative if n ≥ 3 (see Exercise 23 §1.4).

Example 5. If X is a nonempty set, let M = {α img α: XX is a mapping}. Then M is a monoid using composition of mappings as the operation and the identity mapping 1X as the unity (Theorem 3 §0.3). Moreover, M is noncommutative if X has at least two elements.

Example 6. Let * be the operation defined on img by n * m = nm. This operation is neither commutative (2 * 3 = 8 but 3 * 2 = 9) nor associative ((2 * 3) * 2 = 64, but 2 * (3 * 2) = 512), and there is no unity (m = x * m for all m is impossible). Thus img is not a monoid. Note, however, that m * 1 = m for all m.

A comment on notation is in order here. Binary operations are denoted by many different symbols in mathematics. For example, + and · are universally used for addition and multiplication of numbers, but these symbols are also standard for the addition and multiplication of matrices. Similarly, ∩ and ∪ are well-established notations in set theory. When a binary operation has such a standard symbol, we use it along with any standard notation for the corresponding unity (as in the foregoing examples). However, when discussing monoids in general, we have been using * for the binary operation. But algebraists do not do this. They usually adopt one of the following two formats.

  • Multiplicative Notation. Here a * b is written as ab (or sometimes a · b)  and is called the product of a and b. The multiplicative unity is denoted 1  (or 1M if the monoid M must be emphasized).
  • Additive Notation. Here a * b is written as a + b and is called the sum  of a and b. The additive unity is denoted 0 (or 0M if the monoid M must  be emphasized).

Multiplicative notation is the most popular format among algebraists. Hence we adopt the following convention.

Convention. In dealing with monoids in general, we use multiplicative notation, and denote the unity by 1.

Hence ab can mean many different things, depending on the monoid under discussion, but the meaning is nearly always clear from the context. The small amount of confusion is more than balanced by the simplicity and conciseness of the notation.

For a finite monoid M, defining the operation by means of a table is sometimes convenient (as in Example 7 below). Given x and y in M, the product xy is the entry of the table in the row corresponding to x and the column corresponding to y. Hence, for the table in Example 7, ab = b and ca = e. The elements of the monoid appear in the same order across the top of the table as down the left side. Such a table is called the Cayley table of the monoid, honoring Arthur Cayley who used it in 1854.

Example 7. If M = {e, a, b, c}, consider the binary operation shown in the table. The first row and column show that e is the unity. That the operation is commutative is also clear from the table because the entries are symmetric about the main diagonal (upper left to lower right). However, this operation is not associative. For example img

img

If a, b, c, and d are elements in a monoid M, there are various ways to form the product abcd—for example [(ab)c]d and a[b(cd)]. Verifying that these forms are equal is not difficult using associativity. In fact, we have

Theorem 2. General Associativity. Let a1, a2, img, an be elements of a monoid M. If the product a1a2 img an is formed (in that order), the result is the same no matter which bracketing is used.

Proof20. Let the standard product img be defined inductively by setting img and img for n ≥ 2. Thus, img and so on. We use strong induction on n ≥ 1 to prove the following statement: If p is any product of a1, a2, img, an in that order, then img This is clear if n = 1 or n = 2; if n = 3, the only non-standard product is (a1a2)a3, which equals img by associativity. In general, because p is formed using multiplication, it must factor as p = qr, where q is a product of a1, a2, img, ak and r is a product of ak+1, img, an for some k with 1 ≤ kn − 1. Hence img by induction. If k = 1, then

img

as required. If k > 1, then img by induction, and

img

which completes the proof.

Theorem 2 enormously simplifies notation. It means that, in a monoid, we may (and do) write a1a2 img an for the product of n elements with no ambiguity. If the operation were not associative, we would have to be careful about which bracketing we use. Of course, the order of the factors in a product does make a difference if the operation is not commutative.

Let a be an element of a monoid M. If n ≥ 0 is an integer, inductively define the nth power an of a as follows:

img

Thus, a1 = a, a2 = a · a, a3 = a · a · a, and so on. The following laws, familiar for numbers, hold for any monoid.

Theorem 3. Exponent Laws. Let a and b be elements of a monoid M.

1. anam = an+m for all n ≥ 0 and m ≥ 0.

2. (an)m = anm for all n ≥ 0 and m ≥ 0.

3. If ab = ba, then (ab)n = anbn for all n ≥ 0.

Proof. (1) Fix m ≥ 0 and prove (1) by induction on n ≥ 0. If n = 0 then a0am = 1am = am = a0+m. If n ≥ 1 then anam = (aan−1)am = a(an−1am). Since an−1am = an−1+m by induction, this gives anam = a(an−1+m) = an+m.

(2) Fix n ≥ 0 and induct on m using (1). If m = 0, then (an)0 = 1 = an·0. If m ≥ 1, then (an)m = an · (an)m−1 = anan(m−1) = an+n(m−1) = anm.

(3) This assertion follows by induction on n after first showing ban = anb for all n ≥ 0 (Exercise 10).

It is interesting to note that, in the monoids of Example 3 (with ∩ and ∪ as the operations), a2 = a for all a. Hence an = a for all n ≥ 1.

Inverses

If s is a nonzero real number, the inverse img is the solution to the equation xs = 1. In this form the idea extends to any monoid. If a is an element in a monoid M, an element b of M is called an inverse of a if ab = 1 = ba. An element with an inverse is called a unit. Note that the definition is symmetric in a and b, so that a is an inverse of b if and only if b is an inverse of a.

Theorem 4. If M is a monoid and a img M has an inverse in M, then that inverse is unique .

Proof. If both b and b′ are inverses of a, then ab = 1 = ba and ab′ = 1 = ba. Hence b′ = b′1 = b′(ab) = (ba)b = 1b = b.

Note the use of associativity in Theorem 4. In fact, its use is essential: In Example 7, both a and c are inverses of c.

If a is a unit in a multiplicative monoid, the inverse of a is denoted a−1. If the monoid is additive the inverse of a is denoted −a and is called the negative of a.

Example 8. Consider the additive monoids img and img Then every element is a unit and, in all cases, the usual negative −x of an element x is the (additive) inverse.

Example 9. In the multiplicative monoids img and img every nonzero element is a unit. However, 0 has no inverse in either case.

Example 10. The units of img are 1 and −1.

Example 11. The units in img are the residues img where a and n are relatively prime (Theorem 5 §1.3).

Example 12. If M = {α img α: XX is a mapping} under composition, the units in M are the bijections (onto and one-to-one mappings). (See Theorem 6 §0.3.)

Example 12 is important, and we refer to it again. If X = {1, 2, img, n}, the set of units is the symmetric group Sn of degree n (Section 1.4). If img we get a monoid containing maps σ and τ such that στ = 1 but τσ ≠ 1. (Example 8 §0.3.)

Example 13. The units in img are the matrices A with det A ≠ 0, where det A denotes the determinant of A.21 This is discussed in Appendix B.

The next theorem collects several basic properties of units that will be used without comment throughout the book. This theorem will be familiar to students of linear algebra where it is proved for invertible matrices.

Theorem 5. Let a, b, a1, a2, img, an−1, an denote elements in a monoid M.

1. 1 is a unit and 1−1 = 1.

2. If a is a unit so is a−1, and (a−1)−1 = a.

3. If a and b are units so is ab, and (ab)−1 = b−1a−1.

4. If a1, a2, img, an−1, an are units, so is a1a2 img an−1an, and img

5. If a is a unit so is an for any n ≥ 0, and (an)−1 = (a−1)n.

Proof. (1), (2), and (3) depend on the fact that, if ab = 1 = ba for some b then a is a unit and a−1 = b. Thus (1) follows from 1· 1 = 1; (2) follows from a−1a = 1 = aa−1, and (3) follows if we can show that

img

But (ab)(b−1a−1) = a(bb−1)a−1 = a1a−1 = aa−1 = 1. The other equation can be similarly verified.

Finally (4) follows from (3) by induction on n (Exercise 16), and (5) is the special case of (4) where a1 = a2 = img = an = a.

Note that every monoid has at least one unit: the unity. Moreover, if M is the set of all subsets of a set U, then (M, ∩) and (M, ∪) are monoids in which the unity is the only unit. At the other extreme are the monoids (called groups) in which every element is a unit. These are the principal objects of study in this chapter. With this in mind, we extend the definition of nth powers to include negative powers of a unit. Since (a−1)n = (an)−1 by Theorem 5 for any unit a, we define the negative powers an, n ≥ 1, by

img

Then the laws of exponents extend as follows (the proof is left to the reader).

Theorem 6. Let a and b denote units in a monoid M.

(1) anam = an+m for all img

(2) (an)m = anm for all img

(3) If ab = ba, then (ab)n = anbn for all img

2.1 Exercises

1. In each case a binary operation * is given on a set M. Decide whether it is com-mutative or associative, whether a unity exists, and find the units (if there is a unity).

(a) img a * b = ab

(b) img; img

(c) img

(d) M = any set with |M|≥ 2; a * b = b

(e) M = P × Q, where P and Q are sets with |P| ≥ 2 and |Q|≥ 2; (p, q) * (p′, q′) = (p, q′)

(f) img m * n = max (m, n)—the larger of m and n

(g) img

(h) img

(i) img

(j) img

2.

(a) If x, y, or z is 1, show that (xy)z = x(yz).

(b) Show that there are exactly two monoids with two elements.

(c) Let S be a set with an associative binary operation but with no unity. Choose an element 1 ∉ S, write M = {1} ∪ S, and define an operation on M by using the operation of S and 1s = s = s1 for all s img S. Show that M is a monoid.

3. Consider the partial Cayley tables: (1)

a b
a b
b a

 and (2)

a b
a a
b b

(a) Show that there is only one way to complete table (1) so that the resulting op-eration is associative, and that the result makes {a, b} into a commutative monoid.

(b) Show that there are three associative completions of table (2), two making {a, b} into a commutative monoid and one having no unity.

4. If M is any monoid, let img denote the set of all nonempty subsets of M and define an operation on img by XY = {xy img x img X, y img Y}. Show that img is a monoid, commutative if M is, and find the units.

5. Given an alphabet A, call an n-tuple (a1, a2, img, an) with ai img A a word of length n from A and write it (as in English) as a1a2 img an. Multiply two words by (a1a2 img an) · (b1b2 img bm) = a1a2 img anb1b2 img bm, and call this product juxtaposition. Thus, the product of “ no” and “ on” is “ noon”. We decree the existence of an empty word λ with no letters. Show that the set W of all words from A is a monoid, noncommutative if |A| > 1, and find the units.

6. Given a set X and a monoid M, let F = {σ img σ: XM is a mapping}. Given σ and τ in F, define σ · τ: XM by (σ · τ)(x) = σ(x)τ(x). Show that this definition makes F into a monoid, commutative if M is, and find all the units.

7. If M and N are monoids, show that the cartesian product M × N is a monoid (called the direct product of M and N) using the operation (m, n)(m′, n′) = (mm′, nn′). When is M × N commutative? Describe the units.

8. An element e of a monoid M is called an idempotent if e2 = e.

(a) If a img M satisfies am = am+n, where m ≥ 0 and n ≥ 1, show that some power of a is an idempotent. [Hint: am+r = am+kn+r for all k ≥ 1 and r ≥ 0.]

(b) If M is finite, show that some positive power of every element is an idempotent.

9. Assume that a is left cancelable in a monoid M (ab = ac implies that b = c).

(a) If a5 = b5 and a12 = b12 in M, show that a = b.

(b) If am = bm and an = bn where m and n are relatively prime, show that a = b.

10. If ab = ba in a monoid M, prove that (ab)n = anbn for all n ≥ 0 (Theorem 3(3)).

11. An element e is called a left (right) unity for an operation if ex = x (xe = x) for all x. If an operation has two left unities, show that it has no right unity.

12.

(a) If u is a unit in a monoid M, show that au = bu in M implies that a = b.

(b) If M is a finite monoid and au = bu in M implies that a = b, show that u is a unit. [Hint: If M = {a1, img, an} show that a1u, . . ., anu are distinct.]

13. If uv is a unit in a monoid M, and if av = bv implies that a = b in M, show that u and img are both units.

14. If uv = 1, we say that u is a left inverse of img and img is a right inverse of u. If u has both a left and a right inverse in a monoid, show that u is a unit.

15. If M is a monoid and u img M, let σ: MM be defined by σ(a) = ua for all a img M.

(a) Show that σ is a bijection if and only if u is a unit,

(b) If u is a unit, describe the inverse mapping σ−1: MM.

16. If u1, u2, img, un−1, un are units in a monoid, show that u1u2 img un−1un is also a unit and that img (Theorem 5(4)).

17. Let u and img be units in a monoid M.

(a) If img show that img

(b) If a img M and ua = au, show that u−1a = au−1.

(c) If uv = vu, show that img

18. Prove that the following are equivalent for a monoid M.

(1) If ab is a unit then both a and b are units.

(2) If ab = 1, then ba = 1.

19. If M is a finite monoid and uv = 1 in M, prove that vu = 1. [Hint: Exercise 12(b).]

20. Let M be a commutative monoid. Define a relation img on M by a img b if a = bu for some unit u.

(a) Show that img is an equivalence on M.

(b) If img denotes the equivalence class of a, let img denote the set of all equivalence classes. Show that img is a well-defined operation on img

(c) If img is as in (b), show that img is a commutative monoid in which the unity img is the only unit.

21. If M is a monoid, define E(M) = {α: MM img α(xy) = α(x) · y for all x, y img M}. If a img M, define αa: MM by αa(x) = ax for all x img M.

(a) Show that E(M) is a monoid under composition of mappings.

(b) Show that αa img E(M) for all a img M.

(c) If θ: ME(M) is defined by θ(a) = αa for all a img M, show that θ is onto and one-to-one, θ(1) = 1M, and θ(ab) = θ(a) · θ(b) for all a, b img M.

22. Show that there are exactly six monoids M with three elements. If M = {1, a, b}, consider first the case a2 = 1 (then only one multiplication table is possible). If a2 = b, then M = {1, a, a2} is commutative and there are three monoids. Then two more emerge if a2 = a. Note that, although associativity is used to force the multiplication table in every case, the associativity in the resulting table must be checked (Exercise 2(a) is useful).

2.2 Groups

A group is a monoid in which every element has an inverse. Because of its importance, we give the definition in full detail. A set G is called a group if it satisfies the following axioms.

G1 G is closed under a binary operation.

G2 The operation is associative.

G3 There is a unity element in G.

G4 Every element of G has an inverse in G.

The group G is called abelian22 if, in addition, it satisfies

G5 The operation is commutative.

If G is finite, the number |G| of elements in G is called the order of G.

The terminology used for groups (and other algebraic systems, such as monoids) is somewhat careless. Strictly speaking, a group (G, ·) consists of two things: a set G and a binary operation. However, common practice is simply to refer to a group G and not mention the operation. This practice usually causes no difficulty, because the operation in the group in question is understood. We adopt this loose notation because it is much simpler, and also to acquaint the reader with what is in fact used in more advanced books. When clarity is needed, we use terms such as the group (G, +) or the additive group G.

Examples 1–10 indicate the variety of ways that groups can occur, and we refer to many of them later. We leave verification of the axioms to the reader.

Example 1. {1}, {1, − 1}, and {1, − 1, i, − i} are all abelian groups of (complex) numbers under multiplication. Here −1 is self-inverse, and i and −i are inverses of each other.

Example 2. img23 img and img are all abelian groups under multiplication. In each case the inverse of an element a is a−1 = 1/a.

Example 3. The set of complex numbers

img

is a group under complex multiplication. Here e = cos θ + i sin θ, as in Appendix A, and we have eeiimg = ei(θ+img) and (e)−1 = e. The group img is called the circle group because it consists of the points on the unit circle.

Example 4. For n ≥ 1, the group img is a group under complex multiplication, called the group of nth roots of unity. As in Appendix A, we have

img

Clearly, img for all n ≥ 1, and U1, U2, and U4 are displayed in Example 1.

Example 5. The sets img and img are all abelian groups under addition. In each case the unity is 0 and the inverse of x is −x.

Although we write most groups multiplicatively, many important groups are written additively (as in Example 5). Then the unity element is denoted 0 and is called zero, and the inverse of x is denoted −x and is called the negative of x.

Example 6. If img is an additive abelian group with zero img and the negative of img being img We write img in img when no confusion can result.

Henceforth, when we refer to one of the groups img or img we mean the additive group.

Example 7. The set Sn of all permutations of {1, 2, . . ., n} is a group under composition (see Theorem 2 § 1.4), called the symmetric group of degree n.

The group Sn has historical significance because such groups of bijections were among the earliest examples of a group. They were used by Galois in his pioneering work on the theory of equations. In fact Galois was the first to use the term group.

Example 8. We single out S3 for special emphasis. Recall from Section 1.4 that

img

If we denote σ = (123) and τ = (12), then σ2 = (132), τσ = (23), and τσ2 = (13) as is easily verified. Hence we can list S3 as

img

The reason for doing this is that it provides an easy way to fill in the Cayley table. In fact, we can fill in the table by using three (easily verified) facts:

img

The resulting Cayley table is as follows

img

Note that

img

Then, for example, we compute the product (τσ)(τσ2) by

img

The other entries in the table are found in a similar manner (the reader should do this). The elements σ and τ are called generators for S3, and the equations σ3 = ε, τ2 = ε, and στσ = τ are called relations among the generators. We often describe S3 in this way.

Examples 9 and 10 display two other important groups of permutations.

Example 9. The set An of all even permutations in Sn is a group using the operation of Sn, called the alternating group of degree n (Theorem 8 §1.4).

Example 10. Given a (nonsquare) wire rectangle with vertices 1, 2, 3, and 4 as in the diagram, consider the permutations of the vertices induced by moving the rectangle in space (without bending). The 180img-rotations about vertical and horizontal axes (see the diagram) give permutations

img

img

respectively. If we compute their product in S4 we obtain another motion στ = (13)(24) because the composite motion στ is the motion τ followed by the motion σ (the reader should verify this). Note that στ can also be viewed as the 180img-rotation in the plane of the rectangle about its center. Of course, we have another motion τσ, but this is not a new motion because

img

img

is called the group of motions of the rectangle. Such groups of motions are important (for example they arise in the study of symmetries of molecules); we return to them in Section 2.7.

Recall that a set M with an associative operation that has a unity is called a monoid, and that an element u in M that has an inverse u−1 in M is called a unit. A monoid may not be a group, but its units form a group.

Theorem 1. If M is a monoid, the set M* of all units in M is a group using the operation of M, called the group of units of M.

Proof. From Theorem 5 §2.1, if u and img are units, then uv is also a unit (the inverse is img), so M* is closed under the operation of M. The associativity of M* is inherited from M and 1 img M* (in fact 1−1 = 1), so M* itself is a monoid. Finally, if u img M*, then u−1 img M* too (its inverse is u), so M* is a group.

Theorem 1 provides many important examples of groups. For example, the multiplicative groups in Example 2 are img and img

Example 11. If X is a nonempty set, M = {α img α: XX is a mapping} is a monoid under composition and Theorem 6 §0.3 shows that the group M* of units consists of the bijections

img

The bijections XX are called permutations of X, and SX is the permutation group of X. Of course, if X = {1, 2, . . ., n} then SX = Sn.

Example 12. Consider img where img is regarded as a multiplicative monoid. Then Theorem 5 §1.3 gives

img

Hence img if (and only if) p is a prime. Other examples include

img

We refer to these groups frequently.

Example 13. Let R denote img. Then the set Mn(R) of all n × n matrices over R is a monoid using matrix multiplication. The group Mn(R)* of units consists of the invertible n × n matrices over R, that is the matrices such that det A is a unit in R—see Appendix B. It is called the general linear group of degree n over R, denoted GLn(R). Thus

img

If G1, G2, img, Gn are sets, recall that the cartesian product G1 × G2 × img × Gn is the set of all ordered n -tuples (g1, g2, img, gn), where gi img Gi for each i. This set has a natural group structure when the Gi are themselves groups. If G1, G2, img, Gn are groups, their direct product is the set G1 × G2 × img × Gn with the component-wise operation defined by

img

where img is the product in Gi for each i. The routine proof of the next theorem is left to the reader.

Theorem 2. If G1, G2, img, Gn are groups, so also is G1 × G2 × img × Gn, with unity (1, 1, img, 1) and inverses img

Because groups are monoids, all the properties of monoids presented in Section 2.1 are automatically properties of groups. In particular:

(1) The unity 1 is unique.

(2) The inverse g−1 of an element g is uniquely determined by g.

(3) General associativity holds (Theorem 2 §2.1).

The next theorem restates Theorem 5 §2.1 for units in monoids for reference.

Theorem 3. Let g, h, g1, g2, img, gn−1, gn denote elements of a group G.

1. 1−1 = 1.

2. (g−1)−1 = g.

3. (gh)−1 = h−1g−1.

4. img for all n ≥ 1.

5. (gn)−1 = (g−1)n for all n ≥ 0.

Recall that negative powers of an element g in a group are defined by gk = (g−1)k for k ≥ 1. The next theorem is a restatement of Theorem 6 §2.1.

Theorem 4. Exponent Laws. Let G be a group with elements g and h.

1. gngm = gn+m for all img

2. (gn)m = gn·m for all img

3. If gh = hg, then (gh)n = gnhn for all img

These laws are important and play a prominent role in Section 2.4.

The assumption that every element of a group has an inverse is a very powerful axiom. In particular, it implies the cancellation laws, which we use countless times in this book.

Theorem 5. Cancellation Laws. Let g, h, and f be elements of a group.

1. If gh = gf, then h = f. (left cancellation)

2. If hg = fg, then h = f. (right cancellation)

Proof. If gh = gf, then left multiplication by g−1 gives (g−1g)h = (g−1g)f. Hence 1h = 1f; that is, h = f. This proves (1), and (2) follows similarly.

Note that “ mixed” cancellation is not valid in general: fg = gh does not imply that f = h. For example, in the group S3, we have (12)(13) = (13)(23) so (13) cannot be cancelled.

Example 14. If G is a finite group and g img G, show that gn = 1 for some n ≥ 1.

Solution. The elements g, g2, g3, img in G cannot all be distinct because G is finite. So gm = gm+n for some m ≥ 1 and n ≥ 1. Thus gm · 1 = gm · gn, so 1 = gn by cancellation.

Another consequence of the fact that all elements of a group have inverses is that equations gx = h and xg = h are always solvable.

Theorem 6. Let g and h be elements of a group G.

1. The equation gx = h has a unique solution x = g−1h in G.

2. The equation xg = h has a unique solution x = hg−1 in G.

Proof. If x = g−1h, then gx = gg−1h = 1h = h, so x is indeed a solution in (1). To prove that it is unique, let y also satisfy gy = h. Then gx = gy, so x = y by cancellation. This proves (1), and (2) follows in the same way.

Corollary. Every row (and column) of the Cayley table of a group G contains every element of G exactly once.

Proof. If g img G, the row of the table corresponding to g consists of the elements gx as x ranges over G. This row contains every element h of G because gx = h is solvable for each h, and it contains h only once because the solution is unique. A similar argument applies to columns.

A group is determined completely by its Cayley table: associativity and existence of the unity and inverses, which are demanded by the group axioms, all depend entirely on the operation. Now consider the (multiplicative) group img of units of img and the (additive) group img The Cayley tables are

img

They are the same in the sense that the Cayley table of img becomes that of img if we replace the symbols 1 and −1 by img and img respectively. Thus img and img are the same groups except for notation, and we say that they are isomorphic, or that they are the same up to isomorphism. We discuss this topic in more detail in Section 2.5; for now we prefer to treat the whole matter informally and call two groups isomorphic if they have the same Cayley table except for notation. As a result we can give an application of the Corollary to Theorem 6.

Example 15. Show that, up to isomorphism, there is only one group G of order 1, 2, or 3, and that group can be described in the following manner.

  • If |G| = 1, then G = {1}.
  • If |G| = 2, then G = {1, g}, where g2 = 1.
  • If |G| = 3, then G = {1, g, g2}, where g3 = 1.

In each case the Cayley table is determined by the laws of exponents.

Solution. In each case we show that there is only one way to fill in the Cayley table. Multiplication by 1 is prescribed. If |G| = 1, then G = {1} and the Cayley table is determined. If |G| = 2, let G = {1, g}, where g ≠ 1. The only entry in the Cayley table that is in doubt is whether g2 = g or g2 = 1. But g2 = g is impossible because it implies that g = 1 by cancellation. Hence g2 = 1 and the table is determined.

Turning to the case |G| = 3, write G = {1, g, h}. Then ghg and ghh by cancellation, so we must have gh = 1. Now repeated use of the Corollary to Theorem 6 (beginning with row 2 and column 3) gives the table on the left.

img

In particular, g2 = h, so G = {1, g, g2}, and g3 = gh = 1, as shown in the table on the right. This table is associative, a known realization being the group {ε, (123), (132)} of permutations.

The groups in Example 15 all have the rather special property that every element is a power of a particular element and are called cyclic groups. There exists a cyclic group of order n for every n ≥ 1. Indeed the group Un of nth roots of unity is cyclic of order n. In fact, if we write img then img has order n and img

We discuss cyclic groups in detail in Section 2.4 and treat them informally for now. They occur frequently, and the following generic notation is useful. Given n ≥ 1, the cyclic group of order n is the group Cn of order n:

img

We write img in this case, and the element a is called a generator of Cn. Our insistence that |Cn| = n means that 1, a, a2, . . ., an−1 are distinct elements of Cn.

The Cayley table of Cn is determined completely by the exponent laws and the condition an = 1. In fact, exponents in Cn can be reduced modulo n. That is, if k = qn + r, where 0 ≤ rn − 1, then ak = ar because ak = (an)qar = 1qar = ar. In particular,

img

This expression gives the Cayley table for Cn (below), and so is sufficient for all computations in Cn.

img

Example 16. Let C12 = {1, a, a2, . . ., a11}, a12 = 1, be a cyclic group of order 12. Compute a89 and a−40 in C12.

Solution. Because 89 = 7 · 12 + 5, we get a89 = (a12)7a5 = 17a5 = a5. Similarly, −40 = (− 4) · 12 + 8, so a−40 = (a12)−4a8 = 1−4a8 = a8.

Example 17. Show that bn = 1 for every element b of Cn.

Solution. Write img where an = 1. Then b = ak for some k, so we have

img

Example 15 shows that every group of order 1, 2, or 3 is cyclic. However, this is not the case for groups of order 4.

Example 18. Show that there are only two groups of order 4, the cyclic group C4 and a noncyclic group K4 whose Cayley table is shown below.

img

img

table is the one on the left below. In that case a2 = c, a3 = ca = b, and a4 = c2 = 1, so img is cyclic.

img

img

Similarly if the product of any two of a, b, and c equals 1 then G is cyclic (possibly with a different generator). Thus, if G is not cyclic, the product of any two of a, b, and c must equal the third (for example, bcb, c, or 1, so bc = a). Hence we get the Cayley table on the right as required.

The group K4 = {1, a, b, c} in Example 18 is called the Klein group.24 The multiplication can be described as follows: a2 = b2 = c2 = 1, and the product of any two of a, b, and c is the third.

If you are nervous because we have not shown that K4 is associative, you can relax. The (associative) group img has exactly the Cayley table of K4 if we write a = 3, b = 5, and c = 7. Another instance of K4 is the permutation group K = {ε, (12)(34), (13)(24), (14)(23)}. Example 18 shows that there are two groups of order 4: the cyclic group and the (noncyclic) Klein group. The reader should try to show that every group of order 5 is cyclic; in fact, if p is any prime, we show in Section 2.6 that every group of order p is cyclic.

Exercises 2.2

1. In each case either show that G is a group with the given operation or list the axioms that fail.

(a) img addition

(b) img addition

(c) img

(d) img

(e) G ={ε, (12), (13), (14)}; operation in S4

(f) G ={0, 2, 4, 6}; addition in img

(g) G ={16, 12, 8, 4}; multiplication in img

(h) img multiplication

(i) img is one-to-one}; composition

(j) G ={a, b, c, d}; multiplication given by  

img

2. If G is a group, let Gop denote the set G with a new multiplication given by a img b = ba. Show that Gop is a group.

3. In each case fill in the Cayley table, given that G = {1, a, b, c, d} is a group.

img

img

4. Is the empty set a group? Explain.

5. If M is a monoid, describe an easy way to determine whether M is a group by looking at the Cayley table.

6. If U is a set, let G = {X img XU}. Show that G is an abelian group under the operation ⊕ defined by XY = (X Y) ∪ (Y X).

7. Show that the set img is a group under matrix multiplication.

8. In each case show that G is a group using the operation of S4, and determine how many elements σ of G satisfy σ2 = ε.

(a) G = {ε, (12)(34), (13)(24), (14)(23)}

(b) G = {ε, (1234), (13)(24), (1432)}

9. Let σ = (123456) in S6. Show that G = {ε, σ, σ2, σ3, σ4, σ5} is a group using the operation of S6. Is G abelian? How many elements τ of G satisfy τ2 = ε? τ3 = ε?

10.

(a) If a4 = 1 and ab = ba2 in a group, show that a = 1.

(b) If a6 = 1 and ab = ba3 in a group, show that a2 = 1 and ab = ba.

(c) If a6 = 1 and ab = ba2 in a group, show that a3 = 1 and aba = b.

11.

(a) If (ab)n = 1 in a group where n ≥ 0, show that (ba)n = 1.

(b) Extend (a) to all img

12. Let G be a group of order 4. Assume that 1, a, and b are distinct elements of G and that a2 = 1 and b2 = 1. Show that G = {1, a, b, ab} and fill in the Cayley table.

13. If G is any group, define α: GG by α(g) = g−1. Show that α is onto and one-to-one.

14. Given a, b, and c in a group G, show that the equation a−1xb = c has a unique solution x img G.

15. Let a img G where G is a group. If XG is a finite subset, write Xa = {xa img x img X}. Show that X and Xa have the same number of elements.

16. If fgh = 1 in a group G, show that ghf = 1. Must gfh = 1?

17. Recall that an element e in a monoid is called an idempotent if e2 = e. Describe all the idempotents in a group G.

18. If G is a group and g, h img G, show that gh = hg if and only if g−1h−1 = h−1g−1.

19. Show that a group G is abelian if and only if (gh)−1 = g−1h−1 for all g and h in G.

20. Show that a group G is abelian if g2 = 1 for all g img G. Give an example showing that the converse is false.

21. Show that a group G is abelian if and only if (gh)2 = g2h2 for all g and h in G.

22. Show that a group G is abelian if (gh)3 = g3h3, (gh)4 = g4h4, and (gh)5 = g5h5 for all g and h in G.

23. Let g be an element of a group G. (a) Show that g2 = 1 if and only if g−1 = g. (b) If |G| is finite and even, show that g ≠ 1 in G exists such that g2 = 1.

24. Let a and b be elements of a group G. Prove that (aba−1)k = abka−1 holds for all img (including negative k).

25. If a5 = 1 and a−1ba = bm in a group, prove that img [Hint: Exercise 24.]

26. Show that every cyclic group Cn of order n is abelian.

27. Show that the additive group img is cyclic.

28. Let a and b be elements of a group G. If an = bn and am = bm where gcd (m, n) = 1, show that a = b. [Hint: Theorem 4 §1.2.]

29. Let G be a set with an associative operation defined on it. In each case show that G is a group.

(a) There is a left unity e (eg = g for all g in G), and each element g has a left inverse (hg = e for some h in G).

(b) G is finite and both cancellation laws hold.

(c) Both gx = h and xg = h are solvable in G for all g and h in G.

(d) For all g and h in G, gx = h has a unique solution in G.

30. If G is an abelian group with n elements, show that gn = 1 for every g img G. [Hint: See the proof of Theorem 8 §1.3.]

2.3 Subgroups

Many important groups arise as subsets of known groups. Therefore, we are interested in knowing which subsets H of a group G are themselves groups (with the same operation). Thus a subset H of a group G is called a subgroup of G if H itself is a group using the operation of G. For example, img is a subgroup of img However the multiplicative group img is not a subgroup of img even though img is a subset of img because the operations are different.

Example 1. If G is any group, both {1} and G are subgroups of G. The subgroup {1} is the trivial subgroup of G. Any subgroup other than G is a proper subgroup.

Example 2. Each of the additive groups img is a subgroup of the larger ones.

Example 3. An is a subgroup of Sn.

Example 4. img denotes the circle group, then each of

img

is a subgroup of the larger ones.

In each of these examples, the subgroups of a group G not only have the same operation as G, but they also share the same unity element and the same inverses. This observation is true in general and, in fact, provides a very useful test for when a subset of a group is actually a subgroup.

Theorem 1. Subgroup Test. A subset H of a group G is a subgroup if and only if the following three conditions are satisfied .

(1) 1G img H, where 1G is the unity of G.25

(2) If h img H and h1 img H, then hh1 img H.

(3) If h img H then h−1 img H, where h−1 denotes the inverse of h in G.

In this case, H has the same unity as G and, if h img H, its inverse in H is the same as its inverse in G.

Proof. If H satisfies (1), (2), and (3), then H is closed by (2), the unity of G is the unity for H by (1), and the inverse in G of an element h img H serves as the inverse of h in H by (3). As H inherits the associative law from G, it is a subgroup.

Conversely, if H is a subgroup, let e denote the unity of H. Then e2 = e = e · 1G, so e = 1G by cancellation in G. This proves (1), and (2) follows because H is closed under the operation of G. Finally, if h img H, let h′ denote its inverse in H. If h−1 is the inverse in G, then hh′ = 1 = hh−1, so h′ = h−1 by cancellation in G. This proves (3) and the last sentence in the theorem.

Theorem 1 is useful as the conditions are easily checked (see also Exercise 2).

Example 5. If R is one of img or img let img Show that H is a group using matrix multiplication, called the special linear group.

Solution. We have img —see Example 13 §2.2—so we show that it is a subgroup of img We have I img H because det I = 1. If A and B imgH, then det (AB) = det A det B = 1 · 1 = 1 and det A−1 = 1/det A = 1/1 = 1. These results show that AB img H and A−1 img H, so the subgroup test applies.

Example 6. If n ≥ 0, write img Show that img is a subgroup of img

Solution. The unity of img is 0, and img If a and b are in img write them as a = nk and b = nm, where img and img Then a + b = n(k + m) and −a = n(− k) both lie in img so img is a subgroup of img by the subgroup test.

Theorem 2. Finite Subgroup Test. If H is a finite nonempty subset of a group G, then H is a subgroup of G if and only if H is closed (h, h1 img H implies hh1 img H).

Proof. If H is closed, let h img H. Then each of h, h2, h3, img is in H so, because H is finite, they cannot all be distinct. Hence hn = hn+m for some n ≥ 1 and m ≥ 1. This means 1 = hm by cancellation, so 1 img H by hypothesis. But then 1 = hm−1h implies that h−1 = hm−1, so h−1 img H, too. Because H is closed by hypothesis, it is a subgroup by Theorem 1. The converse is clear.

Example 7. Determine all subgroups of the Klein group K4 = {1, a, b, c}, where a2 = b2 = c2 = 1 and the product of two of a, b, and c is the third.

Solution. Each of Ha = {1, a}, Hb = {1, b}, and Hc = {1, c} is a subgroup by Theorem 2, because a2 = b2 = c2 = 1. Any subgroup H with |H| ≥ 3 must contain two of a, b, and c and so contains the other one (their product). Thus, H = G and the complete list of subgroups is {1}, Ha, Hb, Hc, and G.

Example 8. Determine all subgroups of C4 = {1, a, a2, a3}, a4 = 1.

Solution. Let H = {1, a2}. Then H is a subgroup by Theorem 2 because (a2)2 = a4 = 1. Suppose that K is a subgroup distinct from {1} and H. Then either a img K or a3 img K. If a img K, then (because K is closed) each power a, a2, and a3 is in K, so K = C4. Similarly, H = C4 if a3 img H because, as the reader can verify, C4 = {1, a3, (a3)2, (a3)3}. Thus the subgroups are {1}, H = {1, a2}, and C4.

It is descriptive to draw the lattice diagram of all subgroups of a group G. Here the subgroups are shown in such a way that a line can be drawn up from K to H whenever KH. The diagrams for K4 = {1, a, b, c} and for a cyclic group C4 = {1, a, a2, a3} of order 4 are given below.

img

If G is any group, the center of G is defined26 by

img

The elements in Z(G) are said to be central in G.

Theorem 3. If G is any group, then Z(G)is an abelian subgroup of G.

Proof. Use the subgroup test. Clearly 1 img Z(G). If z img Z(G), then zg = gz for all g img G, so multiplying this equation on the left by z−1 gives g = z−1gz. Then multiplication on the right by z−1 gives gz−1 = z−1g. Thus z−1 img Z(G). Finally, if both y and z lie in Z(G), then, for all g img G,

img

Thus, yz img Z(G), so Z(G) is a subgroup. It is clearly abelian.

Observe that Z(G) = G if and only if G is abelian. At the other extreme, it can happen that Z(G) = {1} so G is as far from abelian as it can be. In fact we have:

Example 9. If n ≥ 3, show that Z(Sn) = {ε}, where ε is the identity permutation.

Solution. If σ img Sn, σε, we must find τ img Sn such that σττσ. Because σε, choose k and m in Xn = {1, 2, img, n} such that σk = mk. Because n ≥ 3, let l, k, and m be distinct, with l img Xn, and take τ to be the transposition τ = (kl). Then (τσ)k = τm = m and (στ)k = σl, so it suffices to show that σlm. But if σl = m then σl = σk, so l = k because σ is one-to-one, a contradiction.

We now turn to two important ways of manufacturing new subgroups from old ones. The straightforward proof of Theorem 4 is left as Exercise 16.

Theorem 4. Let H and K be subgroups of a group G. then their intersection

img

is also a subgroup of G.

Note that HK is a subgroup of both H and K, and is the largest such subgroup in the sense that if X is a subgroup of both H and K then XHK. Incidentally, the union HK of two subgroups is almost never a subgroup (see Exercise 17).

The next theorem introduces another important type of subgroup.

Theorem 5. Let H be a subgroup of a group G. If g img G, then

img

is a subgroup of G. These subgroups are called the conjugates of H in G.

Proof. Clearly, 1 = g1g−1 is an element of gHg−1. Given ghg−1, where h img H,

img

Finally (ghg−1)(gh1g−1) = g(hh1)g−1 for any h, h1 in H, which shows that gHg−1 is closed. Thus it is a subgroup by the subgroup test.

If H is a subgroup of G, then H = 1H1−1, so H is always a conjugate of itself. If H is the only conjugate of H in G (that is, gHg−1 = H for all g img G), then H is said to be self-conjugate (or normal) in G. These subgroups play a fundamental role in group theory, and will be investigated in detail in Sections 2.8,2.9, and 2.10. Example 10 displays a subgroup that is not self-conjugate.

Example 10. Let S3 = {ε, σ, σ2, τ, τσ, τσ2}, where σ3 = ε = τ2 and στσ = τ. Find the conjugates of the subgroup H = {ε, τ}.

Solution. Clearly εHε−1 = H. Since σ−1 = σ2 and στσ = τ, we get

img

Similarly, σ2−2 = {ε, τσ2}. These are all the conjugates of H in G (verify).

Exercises 2.3

1. In each case determine whether H is a subgroup of G.

(a) img

(b) img

(c) img

(d) H = {ε, (123)}, G = S3

(e) H = {ε, (12)(34), (13)(24)}, G = S3

(f) img

(g) img

(h) img

(i) H = {(m, k) img m + k is even}, img

2. If H is a subset of a group G, show that H is a subgroup if and only if H is nonempty and ab−1 img H whenever a img H and b img H.

3. If K is a subgroup of H, and H is a subgroup of G, must K be a subgroup of G? Justify your answer.

4. Let img Show that G = {ε, λ1, λ2, μ1, μ2, μ3} is a subgroup of SX if ε(x) = x, λ1(x) = 1/(1 − x), λ2(x) = (x − 1)/x, μ1(x) = 1/x, μ2(x) = x/(x − 1), and μ3(x) = 1 − x, for all x img X.

5.

(a) If G is an abelian group, show that H = {a img G img a2 = 1} is a subgroup of G.

(b) Give an example where H is not a subgroup.

6.

(a) If G is an abelian group, show that H = {g2 img g img G} is a subgroup of G.

(b) Give an example showing that the converse of (a) is false. (c) Show that H is not a subgroup if G = A4.

7.

(a) If G is a group and g img G, show that img is a subgroup of G.

(b) If G is finite, show that img is a subgroup of G for all g img G.

8. If X is a nonempty subset of a group G, let img be the set of all products of powers of elements of X; more formally

img

(a) Show that img is a subgroup of G that contains X.

(b) Show that img for every subgroup H such that XH.

Thus, img is the smallest subgroup of G that contains X, and is called the subgroup generated by X.

9. If G is a group and g img G, define C(g) = {z img G img zg = gz}. Show that C(g) is a subgroup of G (the centralizer of g in G).

10. Let X ⊆ {1, 2, img, n} be a nonempty set. Show that {σ img Sn img σk = k for all k img X} is a subgroup of Sn.

11. Let img Show that G is a subgroup of img

12. Show that img is a subgroup of img

13.

(a) If G is a group, show that {(g, g) img g img G} is a subgroup of G × G.

(b) Determine the groups G such that {(g, g−1) img g img G} is a subgroup of G × G.

14. If X is an infinite set, let G be the set of all permutations σ in SX such that σx = x for all but a finite number of elements x of X. Show that G is a subgroup of SX.

15. In each case determine all subgroups of G and draw the lattice diagram.

img

16. Let H and K be subgroups of a group G.

(a) Show that HK is a subgroup of G (Theorem 4).

(b) Show that HK is the largest subgroup contained in both H and K in the sense that it contains every subgroup contained in both H and K.

17. If H and K are subgroups of a group G, show that HK is a subgroup if and only if HK or KH.

18. If a and b are real numbers, define img by τa,b(x) = ax + b for all img Show that img a ≠ 0} is a subgroup of img

19. Let H and K be subgroups of a group G and let g img G.

(a) If G is abelian, describe the conjugates of H in G.

(b) Show that (gHg−1) ∩ (gKg−1) = g(HK)g−1.

20.

(a) If H is a subgroup of G and HZ(G), show that H is self-conjugate in G.

(b) Let S3 = {ε, σ, σ2, τ, τσ, τσ2}, where σ3 = ε = τ2 and στσ = τ. Show that H = {ε, σ, σ2} is self-conjugate in G.

21. If img find Z(G).

22. Find img

23. Can a group G have an abelian subgroup not contained in Z(G)? Defend your answer.

24. If ab = ba in a group G, let H = {g img G img agb = bga}. Show that H is a subgroup of G.

25. If H and K are subgroups of G, define HK = {hk img h img H, k img K}. Show that HK is a subgroup if and only if KHHK.

2.4 Cyclic Groups and the Order of an Element

We have already introduced the cyclic groups Cn, n ≥ 1, but discussed these groups only informally. Recall that Cn has the form Cn = {1, a, img, an−1}, where an = 1, so Cn consists of powers of A. In this section, we classify groups consisting of all powers of a particular element and determine all subgroups of such groups. This endeavor is important because these groups are building blocks for all sufficiently “ small” abelian groups (including all finite ones).

We begin by showing that the set of all powers of an element of a group G is an important subgroup of G.

Theorem 1. Let g be an element of a group G and write

img

Then img is a subgroup of G, and img for every subgroup H of G with g img H.

Proof. Clearly, img If img write them as x = gk, y = gm. Then the exponent laws give img and img and the subgroup test applies. Finally, img and if g img H where H is a subgroup then img because gk img H for all integers k.

Hence, if g img G then img is the smallest subgroup of G containing the element g.

If g is an element of a group G, the subgroup img is called the cyclic subgroup of G generated by g. If img for some g img G, we say that G is a cyclic group and that g is a generator of G. Thus, the generic cyclic group Cn = {1, a, img, an−1}, an = 1, is cyclic in the present sense, so the terminology is consistent.

Example 1. If G is any group, img is a cyclic subgroup of G.

Example 2. The group G = {1, − 1, i, − i} is cyclic. In fact, i2 = − 1 and i3 = − i show that img Similarly, img so both i and −i are generators. But −1 is not a generator, because all positive and negative powers of −1 are either 1 or −1. Hence img is not all of G.

If a group X is written additively, recall that the unity element is denoted 0 and the inverse of x img X is denoted −x. The exponent xn (in multiplicative notation) becomes nx here, so the cyclic subgroup generated by x is

img

consisting of the multiples of x. The laws of exponents translate as follows:

img

and if x and y commute

img

Here are two important examples of cyclic additive groups.

Example 3. Show that img is cyclic and that 1 and −1 are the only generators.

Solution. If img then img so img Similarly, img because k = (− k) · (− 1). Clearly img, if n ≠ 1 and n ≠ − 1 (for example img

Example 4. Show that img is cyclic with generator img

Solution. We have img where, for the moment, we revert to the formal img notation for residue classes. Given img in img note that img is a multiple of img and so img It follows that img as required.

Example 5. In the multiplicative group img of nonzero real numbers, the cyclic group img consists of all the powers (positive, zero, and negative) of 3. Note that these powers are all distinct in this case.

Example 6. Consider the group img Here are the powers of 2:

img

If the elements in the bottom row are read left to right they “ cycle” endlessly through the sequence 1, 2, 4 (this is the source of the term cyclic group). Clearly img and the reason that img has three elements is that 3 is the smallest positive integer n such that 2n = 1 in img

Order of an Element

These examples point to one of the most useful concepts in group theory. Let g be an element of a group, and suppose that gk = 1 for some integer k ≠ 0. Since gk = 1 also holds, we may assume that k ≥ 1, so the well-ordering principle guarantees that there is a smallest integer n ≥ 1 such that gn = 1. This integer n is called the order of g, and is denoted o(g) = n. If no such integer n exists we say that g has infinite order and write o(g) = ∞. To sum up:

1. If gk = 1 for some k ≠ 0 then o(g) = n is the smallest integer such that n ≥ 1 and gn = 1.

2. If gk = 1 only if k = 0 then o(g) = ∞.

In particular, in Example 5, o(3) =∞ in img, while in Example 6, o(2) = 3 in img Note that the unity element 1 is the only element of order 1 in any group.

Example 7. Find the order of each element in img Is img cyclic?

Solution. We have o(1) = 1. Since 32 = 9 = 1 in img it follows o(3) = 2. Similarly, o(5) = 2 and o(7) = 2. Hence no element ofimg has order 4, so img is not cyclic.

Example 8. Find img in img

Solution. Write img Then img and img so A6 = I. Since A4I and A5I, we conclude that img

Example 9. If γ = (k1k2 img kr) is a cycle in Sn, then o(γ) = r is the length of γ.

Solution. If the integers k1, k2, . . . kr are uniformly placed on a circle, the cycle γ moves each integer one position clockwise, as shown in the diagram. Hence γ2, γ3, . . . carry each in-teger 2, 3, . . . positions clockwise, respectively, so γnε for 1 ≤ nr − 1, whereas γr = ε. This means that o(γ) = r.

Example 10. Show that o(g−1) = o(g) for any group element g.

img

Solution. If img then (g−1)k = (gk)−1, and it follows that (g−1)k = 1 if and only if gk = 1. Hence the smallest positive integer n (if any) such that gn = 1 is the same as the smallest positive integer n such that (g−1)n = 1. That is, o(g−1) = o(g).

Example 11. If G is a finite group, show that every element g img G has finite order.

Solution. Since G is finite, the powers g, g2, g3, img are not all distinct, so let gk = gm with k < m. Then gmk = 1 where mk > 0 so 0(g) is finite.

Computing the order of an element is simplified by the next theorem

Theorem 2. Let G be a group and let g img G satisfy o(g) = n. Then

1. gk = 1if and only if n|k.

2. gk = gmif and only if km (modn).

3. img where 1, g, g2, img, gn−1 are all distinct.

Proof. We use the laws of exponents.

(1) If n|k, say k = qn, then gk = (gn)q = 1q = 1. Conversely, if gk = 1, write k = qn + r with 0 ≤ r < n (division algorithm). But then we have gr = gk(gn)q = 1(1)q = 1. Since r < n, this contradicts the minimality of n unless r = 0. So r = 0 and n|k.

(2) We have gk = gm if and only if gkm = 1. Now apply (1).

(3) Clearly, img To prove the other inclusion, let img say x = gk. As before, write k = qn + r, where 0 ≤ rn − 1. Then

img

which shows that img Hence img To complete the proof, suppose two of 1, g, g2, img, gn−1 are equal, say gk = gm, where 0 ≤ km < n. Then gmk = 1 and 0 ≤ mk < n. This implies that mk = 0 by the minimality of n, so gm = gk. Thus 1, g, g2, img, gn−1 are distinct.

Theorem 2 asserts that if o(g) = n, then gk = 1 if and only if n|k. The following example illustrates how useful this is.

Example 12. Find the order of 2 in img

Solution. We compute in img: 23 = 8, so 26 = 64 = 7 and 29 = 56 = − 1. Hence 218 = 1, so o(2) divides 18 by Theorem 2. Thus, o(2) is 1, 2, 3, 6, 9, or 18. We have already eliminated 3, 6, and 9, so as 21 = 2 and 22 = 4, the only possibility remaining is o(2) = 18. Note that, since img = 18, this shows that img is cyclic and that 2 is a generator.

The next result is the “ companion” of Theorem 2 for elements g with o(g) = ∞.

Theorem 3. Let G be a group and let g img G satisfy o(g) = ∞. Then

1. gk = 1if and only if k = 0.

2. gk = gm if and only if k = m.

3. img, where the gi are distinct.

Proof. (1) Clearly g0 = 1. If gk = 1, k ≠ 0, then gk = (gk)−1 = 1−1 = 1, too. Hence gn = 1 for some n > 0, which implies that img is finite, contrary to hypothesis. Thus gk = 1 implies that k = 0.

(2) We have gk = gm if and only if gkm = 1. Apply (1).

(3) img by definition, and these powers are distinct by (2).

If o(g) = n, then img too, by (3) of Theorem 2, so img in this case. Since this also holds if o(g) = ∞, we have shown that our two uses of the word “ order” are compatible.

Corollary. We have o(g) = img for every element g of any group.

We now use Theorem 2 to derive an elegant formula for the order of any permutation σ in Sn. Recall that σ factors (uniquely) as a product of disjoint cycles γi (Theorem 5 §1.4). The order of σ turns out to be the least common multiple of the orders of the cycles γi (which are the lengths of the γi by Example 9).

Theorem 4. Let σ be a permutation in St with factorization σ = γ1γ2 img γr into disjoint cycles. Then |σ| = lcm(o(γ1), o(γ2), img, o(γr)).

Proof. Write n = o(σ), ni = o(γi), and m = lcm(n1, n2, img, nr). As ni|m for each i, we have img and so img (because the γi commute). Hence n|m by Theorem 2. To show that m|n, it suffices to show that img for each i (then ni|n by Theorem 2 so m|n by the definition of the least common multiple). We show that img the others are similar. This requires proving that img for all 0 ≤ kn. This is clear if k is fixed by γ1, so let k be moved by γ1. Then k is fixed by each of γ2, img, γr, because the γi are disjoint. Thus, since img we have

img

It follows that img as required.

Example 13. Find the order of

σ = img.

Solution. Here σ = (151013414) (2712611) (39) is the cycle factorization, so Theorem 4 gives o(σ) = lcm(6, 5, 2) = 30.

The next result will be used several times below.

Theorem 5. Let o(g) = n for g in some group. If d|n, d ≥ 1, show that img

Proof. Write img for convenience. Then (gd)k = gn = 1, so we must show that k is the smallest such positive integer. Suppose (gd)r = 1, r ≥ 1. Then gdr = 1 so n|dr by Theorem 2, say dr = qn, q ≥ 1. But then dr = q(dk), so r = qk because these are integers and d ≠ 0. It follows that rk, as required.

Other Properties of Cyclic Groups

Theorem 6. Every cyclic group is abelian, but the converse does not hold.

Proof. Let img be cyclic with generator g. If x, y img G, write x = gk, y = gm, where img Then the exponent laws give

img

so G is abelian. However img is abelian but not cyclic by Example 7.

As the proof of Theorem 6 illustrates, computations in a cyclic group depend entirely on the exponents of the generator. As these exponents are integers, the facts about img derived in Chapter 1 turn out to be useful. In particular, the division algorithm plays a natural role in the proof of Theorem 7.

Theorem 7. Every subgroup of a cyclic group is cyclic.

Proof. Suppose that img is cyclic and let H be a subgroup of G. If H = {1}, then img is cyclic. Otherwise, let gk img H, k ≠ 0. Because H is a subgroup, gk = (gk)−1 img H, and so we may assume that k > 0. Hence let m be the smallest positive integer such that gm img H. Then img and we claim this is equality. To see this, let gk img H and write k = qm + r, 0 ≤ r < m, by the division algorithm. It suffices to show that r = 0 (then img). But gr = (gm)qgk img H, which contradicts the minimality of m unless r = 0.

A cyclic group img can have other generators, for example, img Theorem 8 explicitly describes all generators of a finite cyclic group.

Theorem 8. Let img be a cyclic group, where o(g) = n. Then img if and only if gcd (k, n) = 1.

Proof. If img then img say g = (gk)m, where img Thus g1 = gkm, so n divides 1 − km by Theorem 2. Then 1 − km = qn for img that is, 1 = km + qn, which implies that gcd (k, n) = 1. Conversely, if gcd (k, n) = 1 then 1 = xk + yn for some integers x and y by Theorem 4 §1.2. Hence

img

which implies that img

Hence, for example, if o(g) = 12 the generators of img are the powers gk where gcd (k, 12) = 1, that is g, g5, g7, and g11. In particular, the generators of the additive cyclic group img are the residues img and img

Theorem 9 below gives a complete description of all subgroups of a finite cyclic group G. In particular, it shows that G has a unique subgroup of order k for every divisor k of n, and that these are the only subgroups of G.

Theorem 9. Fundamental Theorem of Finite Cyclic Groups. Let img be a cyclic group of order n.

1. If H is a subgroup of G, then img for some d|n. Hence |H| divides n.

2. Conversely, if k|n, then img is the unique subgroup of G of order k.

Proof. (1) Theorem 7 implies that img for some m. Let d = gcd (m, n); we show that img We have d|m, say m = qd, so img when img On the other hand, d = xm + yn, for some img so

img

Hence img so img But then, img by Theorem 5, so |H| divides n.

(2) Suppose that K is any subgroup of G of order k where k|n. By (1) let img where d|n. Then Theorems 2 and 5 give img It follows that img so img This proves (2).

If G is finite and cyclic and H is a subgroup, part (1) of Theorem 9 shows that |H| divides |G|. In fact, this result is true for any finite group G, cyclic or not. The general result is called Lagrange's theorem, which we prove in Section 2.6.

Example 14. Find all subgroups of C12 and draw the lattice diagram.

Solution. Let img, o(g) = 12. The divisors of 12 are 1, 2, 3, 4, 6, and 12. Using Theorem 5, the unique subgroup of each of these orders is, respectively,

img, img, img, img, img, and img

The lattice diagram is as shown at the right. Note that img if and only if k|m.      img

img

We speak of the cyclic subgroup img as being generated by the single element g. We conclude this section with a brief discussion of subgroups generated by more than one element.

Theorem 10. Let X be a nonempty subset of a group G and let

img

denote the set of all products of powers of (not necessarily distinct) elements of X. Then

1. img is a subgroup of G containing X.

2. If H is a subgroup of G with XH, then img

Proof. (1) Choose x img X (because X ≠ ∅). Then img The set img is clearly closed and, if img is in img then img is also in img Hence img is a subgroup of G by the subgroup test.

(2) If XH and img is in img then each img is in H because xi img XH and H is a subgroup. Hence g img H, proving (2).

Thus, if X is a nonempty subset of a group G, the subgroup img in Theorem 10 is the smallest subgroup of G that contains X (in the sense of (2) of Theorem 10). Hence img is called the subgroup generated by X. If G has the form img for some XG, we call X a set of generators for G; if X is finite, we say that G is a finitely generated group.

Obviously, img so the cyclic groups are exactly the subgroups generated by singleton subsets. Similarly, it is customary to write

img

for finitely generated groups.

Example 15. Consider the symmetric group S3 = {ε, σ, σ2, τ, τσ, τσ2}, where |σ| = 3, |τ| = 2, and στ = τσ2. Then img

Example 16. The Klein group K4 = {1, a, b, ab} is generated by any two nonunity elements.

Exercises 2.4

1. Find all generators of the cyclic group img if

img

2. Find all generators of

img

3. Find all generators of

img

4. In each case determine whether G is cyclic.

img

5.

(a) Is img cyclic? Justify your answer.

(b) Is img cyclic? Justify your answer.

6. If G is a group and g img G, show that img

7. Let o(g)= 20 in a group G. Compute (a) o(g2) (b) o(g8) (c) o(g5) (d) o(g3)

8.

(a) Find an element of maximum order in S5.

(b) Find an element of maximum order in S7.

9. In each case find all subgroups of img and draw the lattice diagram.

(a) o(g) = 8

(b) o(g) = 10

(c) o(g) = 18

(d) o(g)= p3, p is a prime.

(e) o(g) = pq, p and q are distinct primes.

(f) o(g)= p2q, p and q are distinct primes.

10.

(a) If gh = hg in a group and o(g) and o(h) are finite, show that o(gh) is finite.

(b) Show that (a) fails if ghhg by considering img and img

11. Let G be a cyclic group of order n.

(a) Show that gn = 1 for all g img G.

(b) If gm = 1 in G where gcd (m, n) = 1, show that g = 1.

12. Let img in Un. Show that o(g) = n.

13.

(a) If G = {g1, g2, img, gr} is an abelian group, let a = g1g2 img gr. Show that a2 = 1.

(b) Prove Wilson's Theorem: (p − 1) ! ≡ − 1 (modp) if p is a prime. [Hint: img]

14. Suppose that G is a group in which {1} and G are the only subgroups. Show that G is finite and, in fact, is cyclic of order 1 or a prime.

15. Show that img for all a and b in a group G.

16. In each case, find the subgroup img of G.

(a) img is cyclic, x = a4, y = a3

(b) img is cyclic, x = a6, y = a8

(c) img is cyclic, x = am, y = ak, gcd (m, k) = d

(d) G = S3, x = (12), y = (23)

(e) img o(a)= 4 = o(b), x = (a3, b), y = (a, b)

(f) img o(a)= 4, o(b)= 6, x = (a2, b), y = (a, b3)

17.

(a) If XY in a group, show that img

(b) Show that a nonempty subset X is a subgroup if and only if img

18. If img and img show that img

19. If img and xy = yx for all x, y img X, show that G is abelian.

20.

(a) Find three elements of C6 × C15 of maximum order.

(b) Find one element of maximum order in Cm × Cn.

21. Find the smallest positive integer n such that σn = ε for every σ img S5.

22. If σ img Sn and o(σ) = p is a prime, show that σ is a product of disjoint p-cycles.

23.

(a) Show that o(h) = o(ghg−1) for all g, h img G. [Hint: Example 10.]

(b) Show that o(gh) = o(hg) for all g, h img G. [Hint: Example 10.]

24.

(a) If h is the only element of order 2 in a group G, show that h img Z(G). [Hint: Exercise 23(a).]

(b) If a is the unique element of order 3 in G, what can you say about a?

25. Let G and H be cyclic groups, with |G| = m and |H| = n. If gcd (m, n) = 1, show that G × H is cyclic. [Hint: If img and img use Theorem 5 §1.2 to show o((g, h))= mn.]

26. Let o(g) = m and o(h) = n in a group G, where m and n are relatively prime.

(a) If gh = hg, show that o(gh) = mn. Is o(gh) = lcm(m, n) in general? [Hint: Theorem 5 §1.2.]

(b) If o(a) = mn, show that a = gh = hg for some g, h img G with o(g) = m and o(h) = n. [Hint: Theorem 4 §1.2.]

27. Let img be a cyclic group and let img and img be cyclic subgroups.

(a) If o(g) =∞, show that AB if and only if a = qb for some img

(b) If o(g) = n, show that AB if and only if aqb (modn) for some img

28. Let H be a subgroup of a group G and let a img G, o(a) = n. If m is the smallest positive integer such that am img H, show that m|n.

29. If o(g) = n, show that o(gk) = n/d, where d = gcd (n, k). [Hint: Proof of Theorem 9.]

30. Let img where o(g) = n. Given gk img G, show img where d = gcd (k, n). [Hint: Theorem 3 §1.2.]

31. Let img be a cyclic group and let img and img

(a) If o(g) =∞, show that img where m = lcm(a, b).

(b) If o(g) = n, assume (Theorem 9) that a|n and b|n. Show that img where m = lcm(a, b).

32. Show that the following conditions are equivalent for a finite group G.

(1) G is cyclic and |G| = pn, where p is a prime and n ≥ 0.

(2) If H and K are subgroups of G, either HK or KH. [Hint: For (1) ⇒ (2) use Theorem 8.]

33. If a group G has a finite number of subgroups, show that G must be finite.

34. Prove the Chinese Remainder Theorem. Let n1, n2, img, nr be positive integers, relatively prime in pairs. Given integers m1, m2, img, mr, show that there exists img such that mim (modni) for each i. [Hint: Extend Exercise 25 to r groups.]

35.

(a) Let o(a) = m and o(b) = n in a group G. If ab = ba, show that an element c img G exists, with o(c) = lcm(a, b). [Hint: Theorem 10 §1.2, Theorem 8, and Exercise 26(a).]

(b) Let G be an abelian group and assume that G has an element of maximal order n (always true if G is finite). Show that gn = 1 for all g img G. [Hint: Part (a).]

36. Let m be the smallest positive integer such that σm = ε for all σ img Sn. Show that m = lcm(2, 3, 4, 5, img, n).

37. For a deck of 2n distinct cards, a “ perfect shuffle” means cutting the deck into two equal halves and collating them as follows: If the cards were originally in the order 1, 2, 3, 4, img, 2n, they end up in the order 1, n + 1, 2, n + 2, img, n, 2n. In each case, determine the number of perfect shuffles required to bring the deck back into its original order.

(a) n = 4, 5, 6, and 7 (b) n = 8, 9, and 10
(c) n = 12 (d) n = 26 (a regular deck)

2.5 Homomorphisms and Isomorphisms

Mathematicians do not deal in objects, but in relations among objects; they are free to replace some objects by others so long as the relations remain unchanged. Content to them is irrelevant: they are interested in form only.

—Henri Poincaré

Up to this point we have ignored mappings from one group to another. The interesting ones are those that preserve the group multiplication in the following sense: If G and H are groups, a mapping α: GH is called a homomorphism27 if

img

Of course the product ab here is in G while α(a) · α(b) is in H.

Homomorphisms arise in many forms as the following examples illustrate.

Example 1. The mapping img given by α(a) = 3a is a homomorphism of additive groups because α(a + b) = 3(a + b) = 3a + 3b = α(a) + α(b) for all img

Example 2. If a is an element of a group G, define the exponent map img by α(k) = ak for all img Then α is a homomorphism because (as the operation in img is addition)

img

Example 3. Let img denote the group of positive real numbers under multiplication. The absolute value map img given by α(z) = |z| for all img is a homomorphism (in fact, onto) because img for all img

Example 4. Let img denote the general linear group of n × n invertible matrices over img The determinant map img given by A img det A is a homomorphism because det (AB) = det A det B for all matrices A and B, and det A ≠ 0 if A is invertible.

Example 5. The identity map 1G: GG is a homomorphism for any group G because 1G(ab) = ab = 1G(a) · 1G(b) for all a, b in G.

Example 6. For groups G and H, there is at least one homomorphism from G to H, the trivial homomorphism α: GH defined by α(g) = 1 for all g img G.

Example 7. If α: GH and β: HK are homomorphisms, show that the composite map βα: GK is also a homomorphism.

Solution. This is because, for all a and b in G,

img

By definition, a homomorphism α: GH is a mapping that preserves the operation in the sense that α(ab) = α(a)α(b) for all a and b in G. Theorem 1 shows that α is “ structure preserving” in the sense that it also preserves the unity, inverses, and powers.

Theorem 1. Let α: G → H be a group homomorphism. Then

img

Proof.

(1) Here img Now cancel in H.

(2) From (1), α(g−1) · α(g) = α(g−1g) = α(1) = 1, which gives (2).

(3) For k = 0, α(g0) = α(1) = 1 = [a(g)]0. If (3) holds for some k ≥ 0, then

img

Hence (3) holds for all k ≥ 0 by induction. If k < 0, write k = − m, m > 0. Then (2) and the preceding calculation give

α(gk) = α[(gm)−1] = [α(gm)]−1 = [α(g)m]−1 = [α(g)]m = [α(g)]k.

Thus [α(g)]k = α(gk) for all img

Corollary 1. Let α: GH be a homomorphism. If g img G has finite order, then α(g) also has finite order, and o(α(g))divides o(g).

Proof. If o(g) = n then gn = 1, so α(g)n = α(gn) = α(1) = 1. Hence o(α(g)) divides n by Theorem 2 §2.4.

Corollary 2. If α: GH is a homomorphism, write α(G) = {α(g) img g img G}. Then α(G) is a subgroup of H.

Proof. This follows from the subgroup test because of the following observations: 1H = α(1G) img α(G); α(g)α(g1) = α(gg1) img α(G), and α(g)−1 = α(g−1) img α(G).

The group α(G) in Corollary 2 is called the image of α. Note that α: GH is onto if and only if α(G) = H.

Example 8. Let α: GH be an onto homomorphism.

1. If G is abelian show that H is abelian.

2. If img is cyclic show that H is cyclic and img

Solution. Let h, h1 img H. Since α is onto, write h = α(g) and h1 = α(g1), g, g1 img G.

1. If G is abelian: hh1 = α(g)α(g1) = α(gg1) = α(g1g) = α(g1)α(g) = h1h.

2. Let img If h img H, say h = α(g), let g = ak, img It suffices to prove that img But img as required.

Let G and H denote groups. In order to show that two mappings α: GH and β: GH are equal, we must verify that α(g) = β(g) holds for all g img G. However, if α and β are homomorphisms, this need only be checked for all g in some generating set for G—see Theorem 10 §2.4.

Theorem 2. Let α: GH and β: GH be homomorphisms and assume that img is generated by a subset X. Then

img

Proof. If α = β, the condition is obvious. If the condition holds, let g img G and write (Theorem 10 §2.4) img where xi img X and img for each i. Then Theorem 1 gives

α(g) = α(x1)k1α(x2)k2 img α(xn)kn = β(x1)1kβ(x2)2k img β(xn)nk = β(g).

As g img G was arbitrary, this shows that α = β.

Theorem 2 shows that a group homomorphism α: GH is completely determined by its effect on a generating set for G. This is useful because many groups are generated by a relatively small number of elements.

Example 9. Show that there are at most six homomorphisms S3C6.

Solution. As in Example 8 §2.2 we write S3 = {1, σ, σ2, τ, τσ, τσ2} where o(σ) = 3, o(τ) = 2, and στσ = τ, and write img o(c) = 6. Hence img so Theorem 2 shows that a homomorphism α: S3C6 is determined by the choice of α(σ) and α(τ) in C6. Now α(σ)3 = α(σ3) = α(ε) = 1, so o(α(σ)) is 1 or 3. Hence there are three choices for α(σ): 1, c2, or c4. Similarly, α(τ)2 = 1, so α(τ) must be 1 or c3. Thus, there are at most 3 · 2 = 6 choices in all for α(σ).

We hasten to note that not all the choices in Example 9 correspond to actual homomorphisms. In fact, there are only two homomorphisms from S3 to C6, and we return to this example later (see Example 9 §2.10).

Isomorphisms

We have shown that there are two distinct groups of order 4: the cyclic group and the noncyclic Klein group. Determining how to distinguish between distinct groups leads to the notion of isomorphic groups. Roughly speaking, the two groups are isomorphic if they are the same except for notation.

As an illustration, consider the groups G = {1, − 1} and img The two Cayley tables are

img

Clearly, they are alike. In fact, because the way the unity multiplies is always specified, we can describe both by saying that the nonunity element squares to 1. Here is a more precise comparison: Consider the mapping img given by

img

Then α is a bijection, and we can obtain the entire Cayley table for img from that of G by replacing a with σ(a) for every a in G. In other words, the two groups are the same except for notation; we obtain img from G by changing symbols.

This works in general. If G and H are groups and σ: GH is a bijection, we ask when the Cayley table for H results from applying σ to every element of the table for G. Looking at the diagram

img

the required condition is σ(ab) = σ(a)σ(b) for all a and b in G. In other words, σ must be is a homomorphism. This leads to the following definition.

If G and G1 are groups, a mapping

img

if σ is a bijection (one-to-one and onto) which is also a homomorphism. When an isomorphism exists from G to H we say that G is isomorphic to H and we write

img

Hence, if σ: GH is an isomorphism, the group H is just G with the change of notation g img σ(g). As in the preceding illustration, G and H are the same group except for the symbols used. It is useful to think of isomorphic groups as two different realizations of the same (abstract) group.28

Example 10. The set img of even integers is an additive group, in fact a subgroup of img Show that img

Solution. The function img given by σ(k) = 2k for all img is clearly onto, and σ is one-to-one because σ(k) = σ(m) implies 2k = 2m, so k = m. Finally, σ is a homomorphism:

img

for all k and m in img Thus σ is an isomorphism, so img

Note that the argument in Example 10 shows that img for any integer n ≠ 0.

Example 11. If img show that G is a group using matrix multiplication, and that img

Solution. Define img by σ(n) = img for all n in img Then σ is clearly one-to-one, and it is a homomorphism because

img

Hence img is a subgroup of img by Corollary 2 of Theorem 1. Moreover, σ is a bijection img so img is an isomorphism.

Clearly, G img G for any group G (the identity map GG is an isomorphism). However, even though two groups are isomorphic, they sometimes appear to be quite different. As a remarkable example, the group img of all nonzero complex numbers is known to be isomorphic to the circle group img of complex numbers on the unit circle.29 Here is a less spectacular example. Recall that img

Example 12. Show that img where img is additive and img is multiplicative.

Solution. Define img by σ(r) = er, where ex is the exponential function. To show that σ is one-to-one, let σ(r) = σ(s), where img Then er = es so, if ln x denotes the natural logarithm of x, r = ln (er) = ln (es) = s. Thus σ is one-to-one. If img then t > 0, so img and σ(ln t) = elnt = t. Hence σ is onto. Finally,

σ(r + s) = er+s = eres = σ(r) · σ(s), for all r and s in img

which shows that σ is an isomorphism.

Example 13. If G img G1 and H img H1, show that G × G1 img H × H1.

Solution. Let σ: GG and τ: HH1 be isomorphisms, and define a mapping μ: G × HG1 × H1 by μ(g, h) = (σ(g), τ(h)). This is a homomorphism because

img

for all (g, h) and (g′, h′) in G × H. The proof that τ is onto and one-to-one is left to the reader.

Verifying that a particular mapping is an isomorphism requires checking three things: that it is onto; that it is one-to-one; and that it is operation-preserving. Even if a particular mapping α: GH may fail one of these tests, the groups G and H may very well be isomorphic (for example r img r + 1 is a bijection img but it is not an isomorphism). Conversely, showing that G and H are not isomorphic entails showing that no isomorphism exists from G to H. Examples 14 and 15 illustrate this situation.

Example 14. Show that img is not isomorphic to img

Solution. Suppose that img is an isomorphism. Then σ is onto, so let img satisfy σ(q) = 2, and write img Since σ is a homomorphism, we have

img

This is impossible because img (Example 3 §0.1), so no such σ can exist.

Example 15. Let G and H be cyclic groups with |G| = 9 and |H| = 3. Show that G and H × H are not isomorphic, even though both groups have order 9.

Solution. Suppose that σ: H × HG is an isomorphism. If img where o(a) = 9, let a = σ(x) with x img H × H. Then x3 = 1 (this holds in H) so a3 = σ(x3) = 1, a contradiction. Hence img

The reason that img in Example 15 is that, while x3 = 1 for every element x of H × H, this is not the case for G. The condition x3 = 1 for all x is a property of the Cayley table of H × H but not of the Cayley table of G. The fact that group isomorphisms preserve such properties is the reason that H × H is not isomorphic to G. More generally, we can often show two groups are not isomorphic by exhibiting such a property that holds in one but not the other.

Theorem 3. Let G, H, and K denote groups.

1. The identity map 1G: GG is an isomorphism for every group G.

2. If σ: GH is an isomorphism, the inverse mapping σ−1: HG is also an isomorphism.

3. If σ: GH and τ: HK are isomorphisms, the composite map τσ: GK is also an isomorphism.

Proof. (1) is clear.

(2) The inverse mapping σ−1: HG exists because σ is a bijection, and σ−1 is also a bijection (Theorems 5 and 6 §0.3). So it remains to show that σ−1 is a homomorphism. If g1 and h1 are in G1, write g = σ−1(g1) and h = σ−1(h1). Then σ(g) = g1 and σ(h) = h1, so

σ−1(g1h1) = σ−1[σ(g) · σ(h)] = σ−1[σ(gh)] = gh = σ−1(g1) · σ−1(h1).

Therefore σ−1 is a homomorphism, and hence is an isomorphism.

(3) The map τσ is a bijection by Theorem 3 §0.3; now apply Example 7.

Corollary 1. The isomorphic relation img is an equivalence for groups. That is

1. G img G for every group G.

2. If G img H then H img G.

3. If G img H and H img K then G img K.

Proof. (1), (2), and (3) follows from the corresponding items in Theorem 3.

As an illustration of Corollary 1, we show that if G and H are both cyclic of order n then G img H. Indeed img and img by Example 13, so G img H by Corollary 1.

Automorphisms

If G is a group, an isomorphism GG is called an automorphism of G.

Corollary 2. If G is a group, the set of all automorphisms GG forms a group under composition.

Proof. The automorphisms GG are a subset of the group SG of all bijections GG, and Theorem 3 shows that they are a subgroup of SG by the subgroup test.

The set of all automorphisms of G is called the automorphism group of G, and is denoted autG.

Example 16. If G is abelian, the mapping σ: GG defined by σ(g) = g−1 for all g img G is an automorphism of G. We leave the verification as Exercise 10.

If G is a group and a img G, define a mapping σa: GG by

img

This map σa is an automorphism of G (see Example 17 below), called the inner automorphism of G determined by a. Note that if HG is a subgroup then σa(H) = aHa−1 is a conjugate of H.

Example 17. If G is any group and a img G, show that

1. For each a img G, σa is an automorphism of G.

2. If θ: G → autG is defined by θ(a) = σa for each a img G, then θ is a homomorphism, that is σab = σaσb for all a, b img G.

3. The image θ(G) = {σa img a img G} of θ is a subgroup of autG.

Solution. (1) We leave as Exercise 11 the verification that σa is a bijection for all a img G. If g, h img G we have

σa(g) · σa(h) = aga−1 · aha−1 = ag1ha−1 = agha−1 = σa(gh).

Hence σa is an automorphism of G, proving (1).

(2) We must show that σaσb = σab for a, b img G. But for any g img G:

σaσb(g) = σa(bgb−1) = a(bgb−1)a−1 = (ab)g(ab)−1 = σab(g).

(3) This follows from (2) and Corollary 2 of Theorem 1.

In Example 17, the group θ(G) of all inner automorphisms G is denoted

img

The group innG is an important subgroup of autG, and it is easily described because each inner automorphism σa is given explicitly in terms of a. By contrast, the group autG can be difficult to determine. We do one simple case in Example 18 below.

Because it is a homomorphism, every isomorphism preserves the unity, inverses, and powers. But isomorphisms also preserve the order of an element (compare with Corollary 1 of Theorem 1).

Theorem 4. Let σ: GG1 be an isomorphism. Then o(σ(g)) = o(g) for all g img G.

Proof. It suffices to show that gk = 1 if and only if [σ(g)]k = 1. If gk = 1, then [σ(g)]k = σ(gk) = σ(1) = 1 by Theorem 1. Conversely, if [σ(g)]k = 1, we have σ(gk) = [σ(g)]k = 1 = σ(1), so gk = 1 because σ is one-to-one.

Example 18. If G is cyclic of order 6, show that autG = {1G, λ}, where λ(g) = g−1 for all g img G.

Solution. Both 1G and (as G is abelian) λ are automorphisms of G. If σ: GG is any automorphism, we show σ = 1G or σ = λ. Write img where o(a) = 6. Theorem 2 shows that the choice of σ(a) completely determines σ. By Theorem 4, we have o(σ(a)) = o(a) = 6, so σ(a) = a, or σ(a) = a5 = a−1. If g img G, write g = ak for some img, so that

img

If σ(a) = a, this gives σ(g) = ak = g for all g img G, that is σ = 1G. If σ(a) = a−1, it shows that σ(g) = (a−1)k = (ak)−1 = g−1 for all g img G, that is σ = λ.

Cayley's Theorem

We conclude this section with a proof of a theorem of Cayley (proved in 1878) that every finite group is isomorphic to a group of permutations. If X is a nonempty set, recall that SX denotes the group of all permutations of X (bijections XX) under composition. We need one simple observation about these permutation groups: If a bijection σ: XY exists then SX img SY. Indeed, if λ img SX we have

img

so σλσ−1 img SY. But then img: SXSY given by img(λ) = σλσ−1 is an isomorphism, as can be readily verified. In particular, SX img Sn whenever |X| = n.

Now let G be a group. We noted earlier that each row of the Cayley table of G is a permutation of G in the sense that each element appears exactly once. Since the row of a img G is {ag img g img G}, this is just the assertion that g img ag is a bijection GG. This is the connection that Cayley noticed between the groups G and SG.

Theorem 5. Cayley's Theorem. Every group G of order n is isomorphic to a subgroup of Sn.

Proof. By the preceding discussion, there is an isomorphism θ: SGSn. So if we can find a one-to-one homomorphism σ: GSG, then G img θσ(G) ⊆ Sn because θσ: Gθσ(G) is an isomorphism, and Cayley's theorem follows.

If a img G, define μa: GG by μa(g) = ag for all g img G. Then it is easy to verify that μa is a bijection (so μa img SG). Hence define θ: GSG by σ(a) = μa for all a img G. Then θ is a homomorphism because μab = μaμb for all a, b img G (verify). Finally, θ is one-to-one because μa = μb implies that a = μa(1) = μb(1) = b. So σ is a one-to-homomorphism, as required.

Cayley's theorem shows that every abstract group of order n is (up to isomor-phism) a subgroup of Sn. Hence, to study the groups of order n, we need only study the symmetric group Sn. At first this approach seems to be an advantage because Sn consists of concrete mappings that can be analyzed using tools (such as cycle factorization and parity) not available in an abstract group. However, these symmetric groups are extremely large, so a subgroup of order n is lost in Sn (for example, |S10| = 10 ! = 3, 628, 800). However, in Section 8.3 we give a generalization of Cayley's theorem that cuts down the size of the symmetric group and so provides more information about G.

Arthur Cayley (1821–1895)  Cayley showed his mathematical talent at an early age, quickly excelling at school. After some initial reluctance, his merchant father sent him to Cambridge at the age of 17. During the following 8 years he read the works of the masters and published more than 20 papers on topics that would occupy him for the rest of his life. In addition, he developed broad interests in literature (he read Greek, German, and French, as well as English), architecture, and painting (he demonstrated talent in watercolors) and became an enthusiastic hiker and mountaineer.

At the age of 25, with no position as a mathematician in view, he began legal training and was admitted to the bar three years later. He earned a comfortable living as a lawyer but resisted the temptation to make a lot of money so as to free himself to do mathematics. And do it he did, publishing nearly 300 papers in 14 years. Finally, in 1863, he accepted the Sadlerian professorship at Cambridge and remained there for the rest of his life, valued for his administrative and teaching skills, as well as for his scholarship.

Although Cayley introduced the concept of an abstract group, his main accomplishments lay elsewhere. With his lifelong friend J. J. Sylvester, he founded the theory of invariants; he was one of the first to consider geometry of more than three dimensions; and he initiated matrix algebra. He also wrote on quaternions, the theory of equations, dynamics, and astronomy. He continued working until his death, leaving 966 papers filling 13 volumes of 600 pages each.

Exercises 2.5

1. In each case show that α is a homomorphism and decide if it is onto or one-to-one.

(a) img given by img for all r in img

(b) α: GG × G given by α(g) = (g, g) for all g in the group G.

2. If G = G1 × G2 is a direct product of groups, define π1: GG1 and σ1: G1G by π1(g1, g2) = g1 and σ1(g1) = (g1, 1). Show that π1 is an onto homomorphism (called the projection of G onto G1), and σ1 is a one-to-one homomorphism (called the injection of G1 into G).

3. If G is any group, define α: GG by α(g) = g−1. Show that G is abelian if and only if α is a homomorphism.

4. If img is fixed and G is an abelian group, define α: GG by α(a) = am for all a img G. Show that α is a homomorphism.

5. Let σa be the inner automorphism of G determined by a. Show thatσa = 1G if and only if a img Z(G).

6. Show that there are exactly two homomorphisms α: C6C4. [Hint: Example 9.]

7. If n ≥ 1, give an example of a group homomorphism σ: GG1 and an element g img G such that o(g)=∞ but o(α(g))= n.

8.

(a) Describe all group homomorphisms img

(b) How many are onto?

9. If α: GG1 is a homomorphism, show that K = {g img G img α(g) = 1} is a subgroup of G (called the kernel of α).

10. Define λ: GG by λ(g) = g−1 for all g img G. Show that λ is a bijection. If G is abelian, show that λ is an automorphism of G.

11. If G is a group and a img G, show that the inner automorphism σa: GG is a bijection.

12. In each case determine whether α: GG1 is an isomorphism. Give reasons.

img

13. Show that img is a subgroup of img isomorphic to {1, − 1, i, − i}.

14. If G is an infinite cyclic group, show that img

15. If img is cyclic with o(a) = n, show that img [Hint: img if and only if ak = am by Theorem 2 §2.4.]

16. Show that img is an automorphism if img for all img (here img denotes the complex conjugate of z).

17. If g and h are elements of a group G, show that img

18. If G is a group of order 2, show that G × G img K4.

19. If σ: GG1 is an isomorphism, show that Z(G1) = σ[Z(G)], where we have σ[Z(G)] = {σ(z) img z img Z(G)}.

20. Write img Show that img whenever n ≠ 0 and m ≠ 0.

21. Show that img is not isomorphic to img

22. Show that img is not isomorphic to img

23. Show that the circle group img is not isomorphic to img

24. Find two nonisomorphic groups of order n2 for any integer n ≥ 2.

25. Are the additive groups img and img isomorphic? Support your answer.

26. Show that img

27. If img and img where o(a)= o(b) = 6, describe all isomorphisms GG1.

28. Show that img where img is the circle group.

29. Define img by τa,b(x) = ax + b for all img and denote img Let img Show that G and G1 are subgroups of img and img respectively, and that G img G1.

30. If img show that G is a subgroup of img and that img

31. In each case, find autG, where img is cyclic of order n: (a) n = 2 (b) n = 3

32. If G is infinite cyclic, determine autG.

33. If Z(G) = {1}, show that G img innG.

34. Given z img Z(G), let Gz denote the set G with a new operation a * b = abz−1. Show that Gz is a group and Gz img G.

35. If G is a group and g img G, let S(g) = {σ img autG img σ(g) = g}.

(a) Show that S(g) is a subgroup of autG for all g img G.

(b) If g1 = τ(g), τ img autG, show that S(g) and S(g1) are conjugate in autG.

36. In a group G, write a img b if b = gag−1 for some g img G (a is conjugate to b).

(a) Show that img is an equivalence relation on G.

(b) Determine which elements of G have singleton equivalence classes.

37. If img and σ: GG1 is an onto homomorphism, show that img where σ(X) = {σ(x) img x img X}.

38. Show that img

2.6 Cosets and Lagrange's Theorem

He [Lagrange] would set to mathematics all the little themes on physical inquiries which his friends brought him, much as Schubert set to music any stray rhyme that took his fancy.

—Herbert Westron Turnbull

In this section we prove one of the most important theorems about finite groups, Lagrange's theorem, which asserts that the order of a subgroup of a finite group G is a divisor of |G|. This has far-reaching consequences as we shall see. The proof involves counting elements of G, and depends on the following basic notion.

Let H be a subgroup of a group G. If a img G we identify two subsets of G:

img

We have H1 = H = 1H, so H is a right and left coset of itself. Also the fact that 1 img H shows a img Ha and a img aH for all a. Of course, if G is abelian then Ha = aH for all a img G and all subgroups H of G. However, this may not hold if G is not abelian (see Example 5 below).

Example 1. Let K4 = {1, a, b, ab} be the Klein group where o(a) = o(b) = 2 and ab = ba. If H = {1, a}, find the cosets of H in K4.

Solution. H1 = H and Ha = {a, a2} = {a, 1} = H too. Similarly, Hb = {b, ab} and Hab = {ab, a2b} = {ab, b} = Hb. Thus, there are exactly two cosets of H in K4: H = {1, a} and Hb = {b, ab} = bH.

Note that the cosets H = {1, a} and {b, ab} form a partition30 of K4. This holds in general and, with the other properties in Theorem 1, makes finding cosets easier.

Theorem 1. Let H be a subgroup of a group G and let a, b img G.

(1) H = H1.

(2) Ha = H if and only if a img H.

(3) Ha = Hb if and only if ab−1 img H.

(4) If a img Hb, then Ha = Hb.

(5) Either Ha = Hb or HaHb = ∅.

(6) The distinct right cosets of H are the cells of a partition of G.

Proof. First, (1) is clear because 1 img H and (2) follows from (3) with b = 1.

(3). If Ha = Hb then a img Ha = Hb, say a = hb, h img H. Hence ab−1 = h img H.

Conversely, suppose that ab−1 img H. Then ha = h(ab−1)b img Hb, so HaHb. But ba−1 = (ab−1)−1 img H too, so HbHa follows in the same way. Hence Ha = Hb.

(4) If a img Hb then ab−1 img H so Ha = Hb by (3).

(5) If HaHb ≠ ∅, we show Ha = Hb. If x img HaHb, then x img Ha so Hx = Ha by (4). Similarly Hx = Hb, so Ha = Hx = Hb. This proves (5).

(6) If HaHb then Ha and Hb are disjoint by (5). In other words, the set of right cosets is pairwise disjoint. Moreover, each a img G belongs to some right coset of H (in fact a img Ha). This gives (6).

Corollary. The analogue of Theorem 1 for left cosets also holds. In particular, (3) becomes aH = bH if and only if b−1a img H. See Exercise 5.

Mnemonic. The condition in (3) that Ha = Hb if and only if ab−1 img H can be remembered by “ right multiplying” Ha = Hb by b−1. Similarly, aH = bH if and only if b−1a img H can be recalled by “ left multiplying” by b−1.

Example 2. Let img where o(a) = 6. Find the right cosets of the subgroups img and img

Solution. We have H = H1 = {1, a3}. Thus a3 img H so H = Ha3 by (4) of Theorem 1. In the same way Ha = {a, a4} = Ha4, and Ha2 = {a2, a5} = Ha5. This exhausts G so the cosets are

H = {1, a3}, Ha = {a, a4}, and  Ha2 = {a2, a5}.

Turning to K we find the partition in one step:

K = {1, a2, a4} and  Ka = {a, a3, a5}.

In Example 2, the cosets of H (and those of K) do indeed partition G into pairwise disjoint cells, as Theorem 1(6) asserts. What is new here is that all the cosets of H have the same number of elements and, similarly, all the cosets of K have the same number of elements. This fact holds in general and lies at the heart of Lagrange's theorem, as we show shortly.

Example 3. Find all the right cosets of the subgroup img in the additive group img

Solution. The notation is additive, so the right coset of img generated by a is img For a = 0, we obtain the coset img itself:

img

Now img so it generates a new coset by Theorem 1:

img

We continue in this way, with 2 and 3 generating new cosets:

img

This is a complete list of cosets, because every integer has the form 4k + r, where the remainder r is 0, 1, 2, or 3.

Example 4. In the group img give a geometrical description of the cosets of the circle group img

Solution. Recall that img is the unit circle. If img then z = re, where r = |z| > 0 and img Hence img. In other words, img is the circle with its center at the origin and radius r = |z|.

All these examples involve abelian groups so Ha = aH always holds. However, this does not hold in general as Example 5 shows.

Example 5. Let G = S3 = {ε, σ, σ2, τ, τσ, τσ2}, where σ3 = ε = τ2 and στσ = τ. Find the right and left cosets of H = {ε, τ}.

Solution. As στ = τσ−1 = τσ2, the cosets are

img

Observe that σH and 2σ2H in this case.

Note that, even though the right and left cosets of H may be different, they all have the same number of elements and there are the same number of them. This holds in general, even when the cosets are infinite sets.

Two sets X and Y are said to have the same cardinality if there is a bijection from one to the other, and in this case we write Of course if the sets are finite this means that they have the same number of elements, and this terminology is sometimes used for infinite sets too.

Lemma. Let H be a subgroup of a group G. Then

1. |H| = |Ha| = |aH| for all a img G.

2. The map Ha img a−1H is a bijection {Ha img a img G} → {bH img b img G}.

Proof. (1) |H| = |aH| since h img ha is a bijection HHa. Similarly, |H| = |Ha|.

(2) We have Ha = Hb img ab−1 img H img a−1H = b−1H by Theorem 1 and its Corollary. So the map in (2) is well defined and one-to-one. It is clearly onto.

Part (2) of the Lemma shows that the sets of right and left cosets of a subgroup H of G have the same number of members (possibly infinite), and this common value has a name:

The index |G: H| of H in G is defined to be the number of distinct right (or left) cosets of H in G.

Note that a subgroup H can be of finite index in G even if both H and G are infinite (for example img by Example 3).

The Lemma enables us to prove the single most important theorem about finite groups: It introduces numerical relations into the theory.

Theorem 2. Lagrange's Theorem. Let H be any subgroup of a finite group G.

(1) Then |H| divides |G|.

(2) The quotient img is the index of H in G.

Proof. Write k = |G: H|, and let Ha1, Ha2, img, Hak be the distinct right cosets of H in G. Then

img

which is a disjoint union by Theorem 1. By the Lemma, |Hai| = |H| for each i, so

img

This proves (1), and it also proves (2) because |G||H| = k = |G: H|.

Note that Lagrange's theorem shows that both the order and the index of a subgroup of a finite group G are divisors of |G|.

Lagrange's theorem has many important consequences.

Corollary 1. If G is a finite group and g img G, then o(g) divides |G|.

Proof. The cyclic subgroup img generated by g has o(g) = |H| by the Corollary to Theorem 3 §2.4. So Lagrange's theorem applies.

Note that the converse of Lagrange's theorem (and of Corollary 1) is false. For example, |A4| = 12, but A4 has no subgroup of order 6 and hence no element of order 6 (Exercise 34).

Corollary 2. If G is a group and |G| = n, then gn = 1 for every g img G.

Proof. If o(g) = m then m|n by Corollary 1, say n = qm for some img But then gn = (gm)q = 1q = 1.

The next corollary will be referred to later, and illustrates how the numerical information in Lagrange's theorem can determine the structure of a finite group.

Corollary 3. If p is a prime, then every group G of order p is cyclic. In fact, img for every element g ≠ 1 in G, so the only subgroups of G are {1} and G.

Proof. Let g ≠ 1 in G and write img Then |H| divides |G| = p, so |H| = 1 or |H| = p. But |H| ≠ 1 because H contains both 1 and g ≠ 1, so |H| = p = |G|. This implies that H = G because G is finite. Finally, if K ≠ {1} is a subgroup of G, and 1 ≠ k img K, then img and so K = G.

Corollary 4. Let H and K be finite subgroups of a group G. If |H| and |K| are relatively prime, then HK = {1}.

Proof. As HK is a subgroup of both H and K, |HK| must divide both |H| and |K| by Lagrange's theorem. Since |H| and |K| are relatively prime, it follows that |HK| = 1. The Corollary follows.

Example 6. Let KHG be finite groups. If |G: K| is a prime, show that H = K or H = G.

Solution. By Lagrange's theorem, img Since |G: K| is a prime, either |G: H| = 1 or |H: K| = 1; that is either H = G or H = K.

We showed earlier (Example 18 §2.2) that every group of order 4 = 22 is either cyclic or is isomorphic to the Klein group. In Example 7, Lagrange's theorem is used to give an analogous result for any prime p in place of 2.

Example 7. If G is a group and img where p a prime, show that either G is cyclic or gp = 1 for every element g img G.

Solution. Assume that G is not cyclic. Then o(g) img p2, so o(g) = 1, p, or p2. But o(g) ≠ p2 because G is not cyclic, so o(g) is 1 or p. Either way gp = 1.

Dihedral Groups

Recall (Example 8 §2.2) that the group S3 can be presented as follows:

S3 = {ε, σ, σ2, τ, τσ, τσ2}, o(σ) = 3, o(τ) = 2, and στσ = τ.

In fact, we can take σ = (123) and τ = (12), but the point here is that the three conditions o(σ) = 3, o(τ) = 2, and στσ = τ are themselves sufficient to fill in the Cayley table of the group.

We now construct a family of groups D2, D3, . . ., Dn, . . . each presented in much the same way as S3, and having D3 img S3. We realize them as subgroups of the group img of 2 × 2 invertible matrices with complex entries.

Let n ≥ 2 be fixed and let img (an nth root of unity). Then img in img Consider the matrices:

img

in img It is easy to verify that o(a) = n, o(b) = 2, and aba = b. This last equation shows that ab = ba−1 = ban−1, and hence that the finite set

img

of matrices is closed under matrix multiplication (I is the 2 × 2 identity matrix). Hence G is a subgroup of img by Theorem 2 §2.3. For convenience, write img Then |A| = n and, as bA, the left cosets A and b A are disjoint. Hence |G| = 2n. We abstract this situation as follows.

If n ≥ 2, the dihedral group Dn is the group of order 2n presented as follows:

Dn = {1, a, a2, . . ., an−1, b, ba, ba2, . . ., ban−1},

 where o(a) = n, o(b) = 2, and aba = b.

Note that the requirement that |Dn| = 2n is equivalent to insisting that img We can carry out all calculations in Dn by using the conditions o(a) = n, o(b) = 2, and aba = b. The equation aba = b implies that akbak = b for all img by induction (Exercise 25), and so we obtain

akb = bak = bank and o(bak) = 2 for all img.

In particular, ab = ban−1, and these formulas enable us to fill in the Cayley table for Dn. Hence the conditions o(a) = n, o(b) = 2, and aba = b completely determine the group Dn (up to isomorphism). The group D4 is called the octic group, and its Cayley table is as follows (the reader should verify this):

img

The group D3 is isomorphic to S3 because

D3 = {1, a, a2, b, ba, ba2}, o(a) = 3, o(b) = 2, and aba = b

which is the same as the presentation of S3 given previously. If n = 2,

D2 = {1, a, b, ba}, o(a) = 2, o(b) = 2, and aba = b.

We have a−1 = a here because o(a) = 2, so D2 is abelian (ba = a−1b = ab) and is isomorphic to the Klein group K4.

Thus, every group of order 4 is either cyclic or dihedral (Example 18 §2.2). The next theorem shows that this result holds for groups of order 2p where p is a prime.

Theorem 3. Let G be a group of order 2p where p is a prime. Then either G is cyclic or G img Dp.

Proof. First, the theorem is true if p = 2 because implies G is cyclic or G img K4 img D2. Hence we assume that p is odd.

Assume that G is not cyclic. Hence o(g) = 1, 2, or p for every g img G by Corollary 1 of Lagrange's theorem. We must show that G img Dp.

Claim 1. G has an element of order p.

Proof. If not, g2 = 1 for all g img G, so G is abelian by Exercise 20 §2.2. Hence if 1, a, and b are distinct in G, then {1, a, b, ab} is a subgroup of order 4 by Theorem 2 §2.3, contrary to Lagrange's theorem. This proves Claim 1.

So let a img G have order p and write img

Claim 2. If x img G and xH, then o(x) = 2.

Proof. We have G = HHx so, because x2Hx, we must have x2 img H. If o(x) = p then, since p is odd, img contrary to the choice of x. Thus o(x) ≠ p, so o(x) = 2 (x ≠ 1 because xH). This proves Claim 2.

Now choose bH. Then G = HbH, a disjoint union, so we obtain

G = {1, a, a2, . . ., ap−1, b, ba, ba2, . . ., bap−1}.

As o(b) = 2 by Claim 2, it remains to show that aba = b. But baH so (ba)2 = 1 again by Claim 2. But then aba = b−1(ba)2 = b−1 = b. Thus G img Dp.

Theorem 3 together with Corollary 3 determines all groups G with |G| ≤ 7:

img

Note that K4 img C2 × C2, so every abelian group here is (isomorphic to) a direct product of cyclic groups. In fact this is true for every finite abelian group, an important result discussed in Chapter 7.

Obviously, the list continues. We will show that there are five nonisomorphic groups of order 8: C8, C4 × C2, C2 × C2 × C2, D4, and another group Q called the quaternion group, to be introduced in Section 2.8. The groups of order 9 are C9 and C3 × C3—both abelian,) and there are two distinct groups of order 10; C10 and D5. The next interesting case is the groups of order 12 (there are five). However, it is not our intention to imply that all the distinct groups of order n have been determined for an arbitrary integer n. That is a very difficult task!

We conclude with an application of Lagrange's theorem to number theory. If n ≥ 2, the Euler function img is defined by

img(n) is the number of integers k img {1, 2, img, n − 1} with gcd (k, n) = 1.

We define img(1) = 1. Hence img(2) = 1, img(3) = 2, img(4) = 2, img(5) = 4, and img(6) = 2. Clearly,

img(p) = p − 1 whenever p is a prime.

Now recall (Theorem 5 §1.3) that, for n ≥ 2, the group of (multiplicative) units in img is given by img and gcd (k, n) = 1}. Hence

img

With this, Lagrange's theorem yields an elegant proof of the following famous result in number theory.

Theorem 4. Euler's Theorem. If a and n ≥ 2 are relatively prime integers, then aimg(n) ≡ 1 (modn).

Proof. We have img Since img Lagrange's theorem (Corollary 2) gives img in img Euler's theorem follows.

A special case gives another proof of Fermat's theorem (Theorem 8 §1.3).

Corollary. Fermat's Theorem. If p is a prime, then ap ≡ a(modp) for all integers a.

Proof. This is clear if a ≡ 0 (modp). Otherwise, a and p are relatively prime so, because img(p) = p − 1, Euler's theorem gives ap−1 ≡ 1 (modp). Fermat's theorem follows.

Joseph Louis Lagrange (1736–1813)  While his name sounds French, Lagrange was born in Italy and spent his early years in Turin. In 1766, he was appointed as Euler's successor at the Berlin Academy by Frederick the Great, who suggested that the “ greatest mathematician in Europe” should be at the court of the “ greatest king in Europe.” After the death of Frederick, Lagrange went to Paris at the invitation of Louis XVI. He remained there throughout the revolution and was made a count by Napoleon who called him the “ lofty pyramid of the mathematical sciences.”

Lagrange was one of the great mathematicians of all time. He made important contributions to many parts of mathematics, including number theory, the theory of equations, differential equations, celestial mechanics, and fluid dynamics. At age 19 he solved a famous problem, the so-called isoperimetrical problem, by inventing an entirely new method, known today as the calculus of variations. His work brought a new level of rigor to analysis. In addition to his mathematical achievements, he was a master of exposition, and his Mechanique Analytique is a masterpiece that William Rowan Hamilton described as a “ scientific poem,”.

In his work on the theory of polynomial equations, Lagrange studied the permutations of the roots of an equation in the hope of finding a general method of solution. He saw that, because the symmetric groups S2, S3, and S4 were sufficiently “ nice” a general solution can always be found if the degree is 2, 3, or 4. But he never discovered what it was about S5 that obstructed the solution of equations of degree 5. Abel, and later Galois, eventually clarified the matter. Nevertheless, Lagrange's work provided one of the sources from which the modern theory of groups evolved.

Exercises 2.6

1. In each case find the right and left cosets in G of the subgroups H and K of G.

(a) img

(b) img

(c) img

(d) img

(e) G = D4 = {1, a, a2, a3, b, ba, ba2, ba3}, o(a) = 4, o(b) = 2, and aba = b; img

(f) G = any group; H is any subgroup of index 2

2. If G is any group, describe the cosets in G of the subgroups {1} and G.

3. If H is a subgroup of G and Ha = Hb where a, b img G, does it follow that aH = bH? Support your answer.

4. If KHG are finite groups, show that |G: K| = |G: H| · |H: K|.

5. If H is a subgroup of G and a, b img G, define ab if b−1a img H.

(a) Show that ≡ is an equivalence relation on G.

(b) Show that the equivalence class (Section 0.4.) of a img G is the left coset aH.

6. Let img with addition (x, y) + (x′, y′) = (x + x′, y + y′). Let H be the line y = mx through the origin: img Show that H is a subgroup of G and describe the cosets H + (a, b) geometrically.

7. Let H be a subgroup of G and suppose that Ha = bH for a, b img G. Show that aH = Hb.

8. Let H and K be subgroups of G. If HaKb for some a, b img G, show that HK.

9. In each case give a geometric description of the cosets of H in G.

img

10.

(a) If img and o(a) = 30, find the index of img in G.

(b) Let img o(a) = n. If d|n, find the index of img in G.

11. Let H and K be subgroups of some group G.

(a) Show that HaKa = (HK)a for all a img G.

(b) Given a, b img G, show that either HaKb is empty or HaKb = (HK)c for some c img G.

12. Let G denote a group and let g img G. In each case show img

(a) |G| = 12, g4 ≠ 1, g6 ≠ 1.

(b) |G| = 40, g8 ≠ 1, g20 ≠ 1.

(c) |G| = 60, g30 ≠ 1, g20 ≠ 1, and g12 ≠ 1.

(d) Generalize. [Hint: Prime factorization.]

13. Let K = {ε, (12)(34), (13)(24), (14)(23)}, and let H be a subgroup of A4 containing K. If H contains any 3-cycle, show that H = A4.

14. Suppose that G has subgroups of orders 45 and 75. If |G| < 400, determine |G|.

15. If H and K are subgroups of a group and |H| is prime, show that either HK or HK = {1}.

16. Let G be a group of order n and let m be an integer with gcd (m, n) = 1. (a) If gm = 1 in G, show that g = 1. [Hint: Theorem 4 §1.2.] (b) Show that each g img G has an mth root, that is that g = am for some a img G.

17. Let |G| = p2, where p is a prime. Show that every proper subgroup of G is cyclic.

18. Let |G| = p3, where p is a prime. If G is not cyclic, show that img for all g img G.

19. Let ak = bk in a group. If o(a)= m and o(b)= n, where n and m are relatively prime, show that mn divides k. [Hint: Lagrange Corollary 4 and Theorem 5 §1.2.]

20. Show that img is even if n ≥ 3. [Hint: Corollary 1 of Lagrange's theorem.]

21. Show that img for every n ≥ 1.

22. If G is a group of order n, define σ: GG by σ(g) = gm for all g img G. If gcd (m, n) = 1, show that σ is a bijection (an automorphism if G is abelian).

23. If G is a group of order pk, where p is a prime and k ≥ 1, show that G must have an element of order p. [Hint: Theorem 5 §2.4.]

24. If G is a group of order pq, where p and q are primes, show that every proper subgroup of G is cyclic.

25.

(a) In Dn, show that akbak = b for all img

(b) In Dn, show that o(bak)= 2 for all img

26. If n ≥ 3, show that Z(Dn) = {1} if n is odd, and that Z(D2m) = {1, am}.

27. Is D5 × C3 img D3 × C5? Prove your answer.

28. If k|n, k ≥ 2, show that Dn has a subgroup isomorphic to Dk.

29. Let G be a group and let p be a prime.

(a) If H and K are subgroups of order p, show that H = K or HK = {1}.

(b) If H1, H2, img, Hk are distinct subgroups of order p, show that

|H1H2imgH2| = 1 + k(p − 1).

(c) If |G| = 15, show that G must have an element of order 3.

30. Let G be any group (possibly infinite) that has no subgroups except {1} and G. If |G| ≥ 2, show that G is finite and cyclic and that |G| is prime. (Converse of Corollary 3 of Lagrange's theorem.)

31. Let KHG be groups. Show that both |G: H| and |H: K| are finite if and only if |G: K| is finite, and then |G: K| = |G: H||H: K|.

[Hint: If |H: K| = n, let Kh1, Kh2, img, Khn be the distinct cosets of K in H. Show that Hg = Kh1gKh2gimgKhng is a disjoint union for all g img G.]

32. Let H and K be subgroups of a group G with |G: H| = m and |G: K| = n.

(a) Show that |G: HK| ≤ mn. [Hint:(HK)g = HgKg for all g img G.]

(b) If gcd (m, n) = 1, show that |G: HK| = mn. [Hint: Exercise 31.]

33. Prove Poincaré 's Theorem: If H1, H2, img, Hn are subgroups of a group G of finite index, then H1H2imgHn is also of finite index. [Hint: Exercise 32.]

34. Show that A4 has no subgroup of order 6, and hence that the converse of Lagrange's theorem is false. [Hint: Theorem 3.]

35. If H and K are subgroups of a group G, define a relation ≡ on G by ab if a = hbk for some h img H and k img K.

(a) Show that ≡ is an equivalence on G.

(b) Describe the equivalence classes (called double cosets).

36. If img is the Euler function, show that n = ∑ d|nimg(d), where the sum is taken over all positive divisors of n. [Hint: Theorems 8 and 9, Section 2.4.]

2.7 Groups of Motions and Symmetries

Group theory began with the study of subgroups of the symmetric group Sn. In this short section we discuss some of these groups, which arise from the symmetries of geometric figures. By a figure we mean a finite set of points called vertices, some pairs of which are joined by line segments. A motion of a geometric figure is a permutation of its vertices that can be realized by a rigid motion in space.

Given two motions σ and τ of a figure, the composite στ is also a motion obtained by first doing τ and then σ. Similarly, σ−1 is a motion achieved by reversing the motion that led to σ. Finally, the identity permutation ε is a motion (resulting from doing nothing at all). Hence the subgroup test gives Theorem 1.

Theorem 1. The set of motions of a figure with n vertices is a subgroup of Sn.

This theorem leads to many interesting groups.

Example 1. Find the group of motions of a (nonsquare) rectangle.

img

Solution. Label the vertices as shown. Then the motions (1 2)(3 4) and (1 4)(2 3) result from rotating the rectangle π radians (180img) about the vertical and horizontal axes of symmetry, respectively. The composite of these is (1 3)(2 4), which is the motion obtained by a rotation of 180img in the plane of the rectangle. Hence

G = {ε, (12)(34), (14)(23), (13)(24)}

is the group of motions. This group is isomorphic to the Klein group.

Example 2. Find the group of motions of an equilateral triangle.

img

Solution. Label the vertices as shown. The motions σ = (1 2 3) and σ2 = (1 3 2) are achieved by clockwise rotations of 2π/3 radians (120img) and 4π/3 radians (240img), respectively. In addition, τ = (1 2) is realized by rotating the triangle π radians (180img) about the line through vertex 3 and the midpoint of the opposite side. Similarly (1 3) and (2 3) are motions, so the group is

S3 = {ε, (1 2 3), (1 3 2), (1 2), (1 3), (2 3)}.

This shows that the equilateral triangle is highly symmetric because every possible permutation of the vertices can be obtained by a rigid motion in space.

It is vital that the rigid motions allowed in Example 2 are rigid motions in space. If the only motions allowed were those in the plane of the triangle, the group of motions would be {ε, (1 2 3), (1 3 2)}. The permutations (1 2), (1 3), and (2 3) cannot be achieved by rigid motions in the plane of the triangle.

A striking illustration of this phenomenon results when we consider the group G of motions of a tetrahedron. This figure is three-dimensional, with four vertices and six edges of equal length as in the diagram. Clearly (123) is a motion of the tetrahedron, obtained by a rotation of 2π/3 radians (120img) about a line through vertex 4 and the center of the opposite face. Similarly, all 3 cycles are in G. The three permutations (1 2)(3 4), (1 3)(2 4), and (1 4)(2 3) are also motions, so A4G, where A4 is the alternating group of all even permutations:

img

img

We claim that A4 = G. Suppose on the contrary that σ img G is an odd motion. If γ = (1 2), write τ = γσ. Then τ is even so τ img G and hence γ = τσ−1 is in G because G is a group. But the transposition γ = (1 2) is not a motion of the tetrahedron, because interchanging vertices 1 and 2 by a rigid motion necessarily interchanges 3 and 4. It follows that

Example 3. The group of motions of the tetrahedron is A4.

This situation is analogous to that for the equilateral triangle in Example 2, where the group of motions is A3 = {ε, (1 2 3), (1 3 2)} if the motions are restricted to the plane containing the triangle. Any odd permutation is achieved as a motion only if the triangle is pulled out of its plane, flipped over, and placed back in its plane. Similarly, no odd permutation of the vertices of a tetrahedron can be realized by a motion in 3-space. It can be achieved only if the figure is “ moved” into 4-space in the process.

Even so, these odd permutations of the vertices of a tetrahedron are symmetries of the figure in the intuitive sense of the word. To make this precise, we let d(x, y) denote the distance between two points x and y in space. As in Section 1.4, if σ img Sn, we write σ(k) = σk for all integers k. Given a geometric figure with n vertices labeled 1, 2, img, n, a symmetry of the figure is a permutation σ of the vertices that preserves the distance between any two vertices; that is

img

Clearly, any motion of a figure is a symmetry, but the converse is not true. For example, the transposition γ = (12) is a symmetry of the tetrahedron, but it is not a motion, as we have demonstrated.

Theorem 2. The symmetries of a figure with n vertices are a subgroup of Sn.

Proof. The identity permutation is clearly a symmetry. If σ and τ are symmetries, then for vertices k and m we have

d[(στ)k, (στ)m] = d[σ(τk), σ(τm)] = d(τk, τm) = d(k, m).

Hence στ is a symmetry. Finally, write σ−1k = k1 and σ−1m = m1. Then k = σk1 and m = σm1 so, since σ is a symmetry,

img

This shows that σ−1 is a symmetry and so completes the proof.

Now let G denote the group of symmetries of the tetrahedron. Then Example 3 gives A4GS4, and A4G because (12) img G. This implies that G = S4 because |S4: A4| = 2 is a prime (see Example 6 §2.6). Hence

Example 4. The group of symmetries of the tetrahedron is S4.

The group of motions (in 3-space) of a geometric figure is thus a subgroup of the group of symmetries, and the two may be distinct as the tetrahedron shows. However, if the figure can be drawn in a plane, the two groups coincide. The reason comes from a theorem of plane geometry. Call a mapping σ from the plane to itself an isometry if it preserves distance; that is if d[σ(x), σ(y)] = d(x, y) for all x and y. It can be shown that every isometry of the plane is a composite of translations, rotations about a point, and reflections in a line. Translations and rotations result from motions in the plane itself, whereas reflections can only be achieved by motions in 3 -space. Thus, every isometry of the plane (and hence every symmetry of a plane figure) is a motion in 3-space. Of course, this condition breaks down for a three-dimensional figure because reflections in a plane are isometries of 3-space that are not motions of 3-space.

We conclude this section by representing the dihedral group Dn as a group of motions. If n ≥ 3, a regular n -gon is a plane figure with n vertices evenly placed on a circle. Thus, a regular 3-gon is an equilateral triangle, a regular 4-gon is a square, and so on. Consider the group G of all motions of a regular n-gon. There are two obvious motions:

(1) σ = (123 img n)—the clockwise rotation of 2π/n radians (360/nimg) about the center of the figure.

(2) τ = (1n− 1)(2n − 2)(3n − 3) img —the rotation of π radians (180img) about a line through the vertex n and the center of the figure.

If n is odd, then τ fixes only the vertex n, whereas if n = 2m, then τ fixes n and m (see Figure 2.1). If λ is any motion of the n-gon, λ is determined by its effect λ1 and λ2 on vertices 1 and 2. If λ2 follows λ1 (clockwise round the n-gon) then λ = σk for some k. On the other hand, if λ2 precedes λ1, then λ = τσk for some k. For example, if n = 7 and the effect of λ is that shown in Figure 2.2, then λ can be achieved by τσ4 as shown in Figure 2.3.

Figure 2.1 The difference when n is odd or even.

img

Figure 2.2 The effect of the motion λ.

img

Figure 2.3 The effect of τσ4 is that of λ.

img

Because |σ| = n and |τ| = 2, it follows that

img

Thus img so |G| = 2n. Moreover, the relation στσ = τ is valid as the following diagram shows.

img

Because |σ| = n, |τ| = 2, and στσ = τ, the definition of Dn (in § 2.6) proves

Theorem 3. The group of motions of a regular n-gon is isomorphic to Dn.

If n = 3, Theorem 3 shows that the group of motions of an equilateral triangle is isomorphic to D3, as is clear from Example 2. If n = 4, it shows that the group of motions of the square is isomorphic to the octic group D4.

Exercises 2.7

1. Find the group of motions of the diamond shown—all edges, and the horizontal diagonal, of length 1.

img

2. Describe a symmetry of the cube that is not a (three-dimensional) motion.

3. Consider the figure where the base edges are of length 1 and the sloped edges are of length 2.

(a) Find the group of (three-dimensional) motions.

(b) Find the group of symmetries.

img

4. Consider the figure where all the edges have length 1 and the base is square.

(a) Find the group of (three-dimensional) motions.

(b) Find the group of symmetries.

img

5. If the double-marked edges of the square shown are painted blue, find the subgroup of the symmetries that carry blue edges to blue edges.

img

6.

(a) Find the group of (three-dimensional) motions of the figure where the triangle edges are of length 1 and the sides are 1 × 2 rectangles.

(b) Find the group of symmetries of the figure.

img

7. Find the groups of motions and symmetries of the figure where each face is a nonsquare rectangle.

img

2.8 Normal Subgroups

If H is a subgroup of a group G, we have seen that aH = Ha may fail to hold for some a img G (Example 2 below). A subgroup H of a group G is called a

normal subgroup of G if gH = Hg holds for all g in G.

In this case H is said to be normal in G, written H img G. These subgroups are of fundamental importance in group theory, and in this section we begin to see why.

Example 1. If G is any group, {1} img G because g{1} = {g} = {1}g, and G img G because gG = G = Gg for all g img G.

Example 2. Let S3 = {ε, σ, σ2, τ, τσ, τσ2}, where o(σ) = 3, o(τ) = 2, and στσ = τ. If H = {ε, σ, σ2} and K = {ε, τ}, show that H img S3 but that K/img S3.

Solution. Clearly, αH = H = for all α img H. Because στ = τσ2 and σ2τ = τσ, we get

= {τ, στ, σ2τ} = {τ, τσ2, τσ} = τH.

Similarly, Hτσ = τσH and Hτσ2 = τσ2H, so H img S3.

However, σK = {σ, στ} and = {σ, σ2τ}, so σK. Hence K/img S3.

Let H be a subgroup of a group G. If g img G satisfies gh = hg for all h img H, then obviously gH = Hg. In particular, this condition holds if each element h of H is in the center Z(G) of G. This proves Theorems 1 and 2.

Theorem 1. If G is a group, every subgroup of the center Z(G) is normal in G. In particular, Z(G) img G.

Theorem 2. If G is an abelian group, every subgroup of G is normal in G.

Note that, given g img G, it is not necessary that gh = hg for all h img H to ensure that gH = Hg. For example, to show that gHHg it is only necessary to show that, given h img H, gh = hg for some himg H.

The converse of Theorem 1 is false: The subgroup H in Example 2 is normal in S3, but H is certainly not central in S3 (in fact, Z(S3) = {ε}). The converse of Theorem 2 is also false: Example 9 below exhibits a nonabelian group in which every subgroup is normal.

Example 3. If K = {ε, (1 2)(3 4), (1 3)(2 4), (1 4)(2 3)}, show that K img A4.

Solution. If σ img Sn and γ = (k1k2 img kr) is a cycle of length r, then σγσ−1 is also a cycle of length r, in fact σγσ−1 = (σk1σk2 img σkr)—see Lemma 3 below. With this, let (ab)(cd) img K. If σ img S4, then

σ[(ab)(cd)]σ−1 = σ(ab)σ−1 σ(cd)σ−1 = (σaσb) (σcσd) img K.

It follows that K img Sn, so certainly Kimg A4.

Theorem 3. Normality Test. The following conditions are equivalent for a subgroup H of a group G.

1. H is normal in G.

2. gHg−1H for all g img G.

3. gHg−1 = H for all g img G.

Proof. (1) ⇒ (2). Let x img gHg−1, say x = ghg−1. Then gh img gH = Hg by (1), say gh = h1g. Then x = ghg−1 = h1gg−1 = h1 img H. This proves (2).

(2) ⇒ (3). If g img G then gHg−1H by (2). Taking g−1 in place of g in (2), we obtain g−1HgH. This implies HgHg−1 (verify), so H = gHg−1, proving (3).

(3) ⇒ (1). Given g img G, we have gHg−1 = H by (3). If x img gH, this shows that xg−1 img H, so x img Hg. This proves gHHg. Since g−1Hg = H by (3) (with g−1 replacing g), a similar argument shows that HggH. Now (1) follows.

Conditions (2) and (3) in Theorem 3 become even more useful if G has a known set X of generators (see Theorem 10 §2.4). The proof is Exercise 13.

Corollary 1. If img, a subgroup H is normal in G if and only if xHx−1H for all x img X. In particular, img if and only if img for all g img G.

If H is a subgroup of G and g img G, recall (Theorem 5 §2.3) that gHg−1 is also a subgroup of G which is isomorphic to H,31 and is called a conjugate of H in G. For this reason, normal subgroups of G are sometimes called self-conjugate subgroups. Incidentally, this discussion proves

Corollary 2. If H is a subgroup of G, and if G has no other subgroups isomorphic to H, then H is normal in G.

In particular, if H is finite and H is the only subgroup of its order, then H img G because |gHg−1| = |H| for all g img G.

Theorem 3 suggests a stronger condition than normality. A subgroup H of a group G is called a characteristic subgroup of G if σ(H) = H for all automorphisms σ: GG (equivalently if σ(H) ⊆ H for all automorphisms σ). The center Z(G) is characteristic in G, and other examples are given in Exercise 24. If σ = σa is the inner automorphism induced by a img G then aHa−1 = σa(H), and it follows that characteristic subgroups are necessarily normal. However, the converse is false by Exercise 24 (c). The following result is often useful.

Corollary 3. If K img G and HK is characteristic in K, then necessarily H img G.

Proof. If a img G, σa: KK is an automorphism of K because K img G. It follows that σa(H) = H because H is characteristic in K. Hence K img G.

Corollary 3 fails if H is merely normal in K (Exercise 4). Many important subgroups are characteristic subgroups (for example, the center); some of their properties are given in Exercise 24.

Example 4. Let img and let H be the subgroup of all matrices with determinant 1. Show that H img G.

Solution. If A img G and B img H, the properties of determinants give

img

This shows that ABA−1 img H, so H is normal in G by part (2) of Theorem 3.

Theorem 4. If H is a subgroup of index 2 in G, then H is normal in G.

Proof. Let a img G. If a img H, then Ha = H = aH. If aH then (because H has exactly 2 right cosets) G = HHa, a disjoint union. Hence Ha = G H. Similarly aH = G H as H has two left cosets, so Ha = G H = aH. Thus, H img G.

Note that subgroups of index 3 need not be normal (Example 2).

Example 5. Show An img Sn where An is the alternating group.

Solution. The alternating group An is of index two in Sn (Theorem 8 §1.4).

Example 6. Let Dn = {1, a, a2, img, an−1, b, ba, ba2, img, ban−1} denote the dihedral group, where o(a) = n, o(b) = 2, and aba = b. Then img by Theorem 4 because img has index 2 in Dn.

We defined H img G to mean gH = Hg for all g img G, which is a kind of commutativity condition on H. The next result gives a situation where actual commuting of elements is implied. It will be referred to later.

Lemma 1. Let H img G and K img G. If HK = {1}, then hk = kh for all elements h img H and k img K.

Proof. Consider x = hkh−1k−1. Thinking of x = h(kh−1k−1) we see that x img H because kh−1k−1 img kHk−1 = H since H img G. Similarly, writing x = (hkh−1)k−1 shows that x img K because K img G. Hence x img HK = {1} by hypothesis, so x = 1. But then hkh−1k−1 = 1, which gives hk = kh.

Let H and K be subgroups of a group G. The intersection HK is the largest subgroup of G contained in both H and K (it contains any such subgroup), and one wonders if there is a smallest subgroup of G containing both H and K. Note that, while HK is the smallest subset containing H and K, it is a subgroup only if HK or KH (see Exercise 17 §2.3). A much more useful construction turns out to be the product HK of the subgroups defined as follows:

HK = {hk img h img H, k img K}.

Then HK contains both H and K, and is contained in any such subgroup, but HK need not be a subgroup (consider H = {ε, τ} and K = {ε, τσ} in S3 with the usual notation). However we do have the following result.

Lemma 2. The following are equivalent for subgroups H and K of a group G:

1. HK is a subgroup of G.

2. HK = KH.

3. KH is a subgroup of G.

Proof. We prove only (1) img (2); then (1) img (3) follows by interchanging H and K.

(1) ⇒ (2). If kh img KH, then kh = (h−1k−1)−1 img HK by (1). This shows that KHHK. On the other hand, if hk img HK then k−1h−1 = (hk)−1 img HK by (1), say k−1h−1 = h1k1. Hence img so HKKH.

(2) ⇒ (1). We use the subgroup test. Clearly 1 = 1 · 1 img HK always holds. If hk img HK then (hk)−1 = k−1h−1 img KH = HK by (2). Finally, given hk and h1k1 in HK, we have kh1 img KH = HK, say kh1 = h2k2. But then it follows that (hk)(h1k1) = h(h2k2)k1 = (hh2)(k2k1) img HK, which completes the proof of (1).

A note of caution is needed here: To say that HK = KH does not mean that hk = kh for all h img H and k img K. To show that HKKH means that, if h img H and k img K are given, then hk = k1h1 for some h1 img H and k1 img K.

If G is abelian then HK is certainly a subgroup by Lemma 2. There is more:

Theorem 5. Let H and K be subgroups of a group G.

1. If H or K is normal in G, then HK = KH is a subgroup of G.

2. If both H and K are normal in G, then HK is also normal in G.

Proof. (1) Suppose that K is normal in G. If hk img HK then hk = (hkh−1)h img KH because hkh−1 img hKh−1 = K. Hence HKKH. The other inclusion is proved the same way, so Lemma 2 applies. A similar argument works if H img G.

(2) If g img G and hk img HK, then g−1(hk)g = (g−1hg)(g−1kg) img HK because H img G and K img G. This proves (2).

Many groups arise as direct products of groups of smaller order, and the following useful theorem gives an important way to recognize when this is the case.

Theorem 6. If H img G and K img G satisfy HK = {1}, then HK img H × K.

Proof. First, HK is a subgroup of G by Theorem 5. Define

σ: H × KHK by σ(h, k) = hk for all h img H and k img K.

We show that σ is an isomorphism. Given (h, k) and (h1, k1) in H × K, we have h1k = kh1 by Lemma 1, so σ is a homomorphism because

σ[(h, k) · (h1, k1)] = σ(hh1, kk1) = hh1 kk1 = hk h1k1 = σ(h, k) · σ(h1, k1).

Since σ is clearly onto, it remains to show that σ is one-to-one. If σ(h, k) = σ(h1, k1), then hk = h1k1 so img Thus img so h = h1 and k = k1. But then (h, k) = (h1k1), proving that σ is one-to-one.

The map σ in Theorem 6 is a bijection for any subgroups H and K. Hence

Corollary 1. If G is a finite group, and H and K are subgroups with HK = {1}, then

Corollary 2. Let G be a finite group and let H and K be normal subgroups such that HK = {1} and |H| |K| = |G|. Then G img H × K.

Proof. By Corollary 1, we have Hence G = HK because HKG and G is finite, so Theorem 6 applies.

Examples 7 and 8 below illustrate how to use Theorem 6. It is easy to verify that the direct product of two cyclic groups of relatively prime orders is again cyclic (Exercise 25 §2.4). Example 7 is the converse. Recall that Cn denotes the generic cyclic group of order n.

Example 7. Let m and n be relatively prime positive integers. If G is a cyclic group of order mn, show that G img Cm × Cn.

Solution. Let img where o(a) = mn, and write img and img Then |H| = o(an) = m and |K| = o(am) = n, so H img Cm and K img Cn. Moreover, HK = {1} by Lagrange's theorem (Corollary 4). Also, H img G and K img G because G is abelian, and |H| · |K| = mn = |G|. Hence G img H × K by Corollary 2 of Theorem 6, that is G img Cm × Cn by Example 13 §2.5.

The fundamental theorem of finite abelian groups asserts that every finite abelian group is isomorphic to a uniquely determined direct product of cyclic groups. We prove this assertion in Section 7.2. Example 8 gives a special case.

Example 8. Let G be an abelian group and assume that img p a prime. Then either img is cyclic or G img Cp × Cp.

Solution. Assume that G is not cyclic. Then o(g) = p if 1 ≠ g img G because o(g) divides p2. Choose a img G with o(a) = p, and write img Then HG, so choose bH, b img G, and write img Hence |K| = p too, so we have img Moreover H img G and K img G because G is abelian. Finally, HK = {1} because, otherwise, H = HK = K by Corollary 3 of Lagrange's theorem. Thus G img H × K by Corollary 2 of Theorem 6, that is Gimg Cp × Cp.

We have already noted (Theorem 2) that every subgroup of an abelian group is normal. The converse is not true: A nonabelian group of order 8 exists in which every subgroup is normal. It is constructed as follows: Let

Q = { ± 1, ±i, ±j, ±k}

be a set of eight elements with multiplication determined by the following equations:

img

Here 1 and −1 multiply as usual, and the multiplication of i, j, and k is best remembered by the diagram above: The product of any two of i, j, and k taken clockwise around the circle is the next one, whereas the product counterclockwise is the negative of the next one. One realization of Q is in img where, if img satisfies img we take

img

The group Q in the preceding discussion is called the quaternion group, and it rules out the converse to Theorem 2.

Example 9. Show that Q is a nonabelian group in which every subgroup is normal.

Solution. Q is nonabelian because ij = k while ji = − k. If a subgroup H contains one of ±i, ±j, or ±k, then |H| = 4 or 8 (because these elements have order 4), so H img Q by Theorem 4. Otherwise, H ⊆ {1, − 1} ⊆ Z(Q) and again H img Q.

Simple Groups

Lagrange's theorem shows that the cyclic groups G of prime order have no subgroups except {1} and G. More generally, if G is a group then we say that

G is simple if G ≠ {1} and the only normal subgroups of G are {1} and G.

Theorem 7. An abelian group G ≠ {1} is simple if and only if it is cyclic of prime order.

Proof. Suppose G is simple and abelian. Then the only subgroups of G are {1} and G (all subgroups are normal). If a img G, a ≠ 1, this means that img Then o(a)≠ ∞ because, otherwise, img does not equal {1} or G. So o(a) is finite, say o(a) = n ≥ 2. If p|n for some prime p, then img is a subgroup of order p by Theorem 5 §2.4. Hence img is cyclic of prime order.

The converse is by Lagrange's theorem.

Nonabelian finite simple groups are more difficult to find. We conclude with a proof that, although A4 is not simple by Example 3, the alternating groups An are all simple if n ≥ 5. This has applications in the theory of equations (Chapter 10). The proof requires three preliminary results, the first two of independent interest.

Lemma 3. If σ img Sn and γ = (k1 k2 img kr) is a cycle of length r, then σγσ−1 is also a cycle of length r. In fact, σγσ−1 = (σk1 σk2 img σkr).

Proof. Write δ = (σk1σk2 img σkr). Because σ is one-to-one, δ is a cycle of length r. We must show that σγσ−1 = δ, that is σγ = δσ, that is σ(γk) = δ(σk) for each k = 1, 2, . . ., n. The reader can verify that (writing kr+1 = k1)

σ(γki) = σki+1 = δ(σki) for each i = 1, 2, . . ., r.

But σ(γk) = σk = δ(σk) whenever k ∉ {k1, k2, img, kr}because (as σ is one-to-one) this implies that σk ∉ {σk1, σk2, img, σkr}. This is what we wanted.

Lemma 4. If n ≥ 2, An is generated by the 3-cycles.

Proof. As each 3-cycle is in An, it suffices to show that each permutation σ img An is either ε or a product of 3-cycles. But σ is even and so is a product of pairs of transpositions. Hence the following formulas complete the proof:

(i j)(i j) = ε, (i j)(i k) = (i k j), and (i j)(k l) = (i l k)(i j k).

Lemma 5. Suppose that n ≥ 5. If H img An and H contains a 3-cycle, then H = An.

Proof. If (i j k) img H we claim that (a j k), (i a k) and (i j a) are all in H for any a ∉ {i, j, k}. Indeed: Since n ≥ 5, choose b ∉ {a, i, j, k}. Then we have (a j k) = (i a b)(i j k)(i a b)−1 img H by Lemma 3. Similarly, (i a k), (i j a) img H.

Let σ = (1 2 3) img H: by Lemma 4 we must show that every 3-cycle τ = (i j k) is in H. Write S = {1, 2, 3} and T = {i, j, k}. If then S = T so τ = σ img H or τ = σ−1 img H. If say i = 1, j = 2 and k ≠ 3, then (1 2 3) img Hτ = (1 2 k) img H by the first paragraph. If say i = 1 and {2, 3} ∩ {j, k} = ∅, then (1 2 3) img H ⇒ (1 j 3) img Hτ = (1 j k) img H. Finally if then (1 2 3) img H ⇒ (i 2 3) img H ⇒ (i j 3) img Hτ = (i j k) img H.

Theorem 8. If n ≥ 5, the alternating group An is simple.

Proof. Let H img An, H ≠ {ε}. Among all elements of H (excluding ε) let τ be one that moves the smallest number m of integers. Then m ≥ 3, because τ img An is not a transposition. If m = 3 then τ is a 3-cycle, and we are done by Lemma 5. So assume m≥ 4; we show that this leads to a contradiction. Factor τ into disjoint cycles and consider two cases.

  • Case 1: τ contains a cycle of length ≥ 3, say τ = (1 2 3 img)γ2 img γr. If τ moves exactly 4 integers, then τ = (1 2 3 k) is odd. So assume that τ moves (say) 4 and 5, as well as 1, 2, and 3. Let β = (3 4 5) and write τ1 = τ−1βτβ−1. Then τ1 img H because H img An, and τ1ε because τ12 = τ−14 ≠ 2. Moreover, if k > 5 is fixed by τ, then k is also fixed by τ1 (because βk = k). Hence if τ1 moves k > 5, then τ also moves k. But τ1 fixes 1, whereas τ does not. Thus τ1 moves fewer elements than τ, a contradiction.
  • Case 2: τ is a product of disjoint transpositions, say, τ = (1 2)(3 4) img. As before, let β = (3 4 5) and τ1 = τ−1βτβ−1. Now τ1 fixes 1 and 2 and any integer k > 5 that is fixed by τ. Because τ1ε (for example, τ15 = 3), this is a contradiction, as in Case 1.

Other infinite families of finite simple groups exist (in addition to the alternating groups An, n ≥ 5). The complete classification of these groups was first given in 1981. This was the culmination of more than 30 years of effort by hundreds of mathematicians, yielding thousands of pages of published work. It is certainly one of the greatest achievements of twentieth-century mathematics. One spectacular landmark came in 1963 when J. Thompson and W. Feit proved32 a long-standing conjecture of William Burnside that every finite nonabelian simple group has even order (the proof is more than 250 pages long!). Thompson went on to publish the “ N -group” paper in which he introduced many fundamental techniques, and which has been called the single most important paper in simple group theory.33 Then in the 1970s, M. Aschbacher carried the work forward in a series of papers, building on the methods of Thompson. The main difficulty was the existence of sporadic finite simple groups not belonging to any of the known families. R. L. Griess finally constructed the largest of these, called the monster (the order is approximately 8 × 1053). The complete classification encompasses several infinite families of finite simple groups and exactly 26 sporadic groups.34

Exercises 2.8

1. Consider D12 = {1, a, img, a11, b, ba, img, ba11}, where o(a) =12, o(b) =2, aba = b. In each case show that H is a subgroup of D12 and determine if H img D12.

(a) H = {1, a6, b, ba6}

(b) H = {1, a4, a8, b, ba4, ba8}

(c) H = {1, a2, a4, a6, a8, a10, b, ba2, ba4, ba6, ba8, ba10}

2. Find all normal subgroups of D4. [Hint: Exercise 7 below.]

3. Let K = {ε, (1 2)(3 4), (1 3)(2 4), (1 4)(2 3)}. Show that K is the only normal subgroup of A4 apart from A4 and {ε}. [Hint: Exercise 34 §2.6.]

4. If D4 = {1, a, a2, a3, b, ba, ba2, ba3}, K = {1, b} and H = {1, a2, b, ba2} show that K img H img D4, but K/img D4.

5. If K img H and H img G, show that aKa−1 img H for all a img G. (See Theorem 5 §2.3.)

6. Let H be a subgroup of a group G. If for each a img G there exists b img G such that aH = Hb, show that H img G.

7. If H img G and |H| = 2, show that HZ(G). Is this true when |H| = 3?

8. If H is a subgroup of G and K img G, show that HK img H. Is HK img K?

9. Given a group G, let D = {(g, g) img g img G}. Show that D is a normal subgroup of G × G if and only if G is abelian.

10. Let N img G and K img G. Show that NK img G.

11. Let p and q be distinct primes. If G is a group of order pq that has a unique subgroup of order p and a unique subgroup of order q, show that G is cyclic. [Hint: Corollary 2 of Theorem 6 and Exercise 25 §2.4.]

12. Let K img G where K is cyclic. Show that every subgroup of K is normal in G.

13. Let X be a nonempty subset of a group G.

(a) If img (see Theorem 10 § 2.4) and H is a subgroup of G, show that H img G if and only if x−1HxH for all x img X.

(b) Show that img is normal in G if and only if img for all g img G.

14. If G = H × K is finite, find H1 img G and K1 img G such that H1H, K1K, H1K1 = {1}, and |G| = |H1| · |K1|. (Converse of Theorem 6.)

15. Let K be a subgroup of G of index 2.

(a) If a img G K and b img G K, show that ab img K.

(b) If H is a subgroup of G and H img K, show that |H: HK| = 2. [Hint: If h0 img H K, show that h img hh0 is a bijection HKH (HK).]

16. Show that innG img autG for any group G.

17. Let Dn = {1, a, img, an−1, b, ba, img, ban−1} with o(a)= n, o(b) = 2, and aba = b.

(a) Show that every subgroup K of img is normal in Dn.

(b) If n is odd and K img Dn, show that K = Dn or img

18.

(a) Let Q denote the quaternion group. If a = i and b = j show that Q has the form Q = {1, a, a2, a3, b, ba, ba2, ba3} where o(a)= 4, aba = b, and b2 = a2. Show further that these conditions determine the Cayley table of Q.

(b) If G is a nonabelian group of order 8, show that GD4 or GQ. [Hint: See Theorem 3 §2.6; use Theorem 4 and (a).]

19. If H and K are subgroups of G, show that HK is a subgroup if and only if HKKH, if and only if KHHK.

20. If G = HK where H and K are subgroups such that hk = kh for all h img H and k img K, show that H img G and K img G.

21. If HG and KG are subgroups with HK = KH, show that img (See Theorem 8 §2.4.)

22. Let G be a group with where m and n are relatively prime, and let H and K be subgroups where If hk = kh for all h img H and k img K, show that GH × K.

23.

(a) Let n = 2m, where m is odd. Show that DnC2 × Dm, where C2 is cyclic of order 2. [Hint: Corollary 2 of Theorem 6.]

(b) Is D12C3 × D4? Justify your answer.

24. A subgroup H of a group G is called a characteristic subgroup if σ(H) ⊆ H for all automorphisms σ of G.

(a) Show that every characteristic subgroup is normal.

(b) Show that if H is characteristic in G then σ(H) = H for all σ img autG.

(c) If G = C2 × C2 show that H = C2 × {1} is normal in G but not characteristic. [Hint: Consider σ: GG given by σ(x, y) = (y, x).]

(d) Show that the center Z(G) is characteristic in G.

(e) If HK img G and H is characteristic in K, show that H img G.

(f) If K is characteristic in H and H is characteristic in G, show that K is characteristic in G. (Compare with Exercise 4.)

(g) Show that every subgroup of a cyclic group G is characteristic in G. Is this true if G is merely abelian?

(h) If H and K are characteristic in G, show that HK is characteristic in G.

(i) If H is a subgroup of G, let K = {g img G img g img σ(H) for all σ img autG}. Show that K is characteristic in G, that KH, and that K contains every characteristic subgroup of G that is contained in H.

25. If X is a nonempty subset of a group G, define the normalizer N(X) of X by N(X) = {a img G img aXa−1 = X}.

(a) Show that N(X) is a subgroup of G.

(b) If H is a subgroup of G, show that H img N(H).

(c) If H is a subgroup of G, show that N(H) is the largest subgroup of G in which H is normal. That is, if H img K, and K is a subgroup of G, then KN(H).

26. If H is a subgroup of G, define the core of H, denoted coreH, to be the intersection of all the conjugates of H in G; that is,

equation

(a) Show that coreH img G and coreHH.

(b) Show that coreH is the largest normal subgroup of G that is contained in H; that is, if K img G and KH, then K ⊆ coreH.

(c) Show that core(HK) = coreH ∩ coreK for all subgroups H and K.

27. If X is a nonempty subset of a group G, define the normal closure img of X to be the intersection of all normal subgroups of G that contain X; that is,

equation

(a) Show that img and img

(b) Show that img is the smallest normal subgroup of G that contains X; that is, XN and N img G implies that img

(c) Show that img for all subgroups H and K of G, and that this need not be equality.

28. If X is a nonempty subset of a group G, define the centralizer C(X) of X by C(X) = {c img G img cx = xc for all x img X}. Note that C(G) = Z(G).

(a) Show that C(X) is a subgroup of G.

(b) If K img G, show that C(K) img G.

2.9 Factor Groups

If n ≥ 2, recall the construction of img in Section 1.3. Given the subgroup img of img the set img consists of all “ residue classes” img (mod n)} where img These classes are really cosets img Moreover, we defined addition in img by img that is,

equation

This suggests a general definition: If K is a subgroup of a multiplicative group G, we could define an analogous multiplication on the set of right cosets by

equation

However, this may not make sense for some subgroups K because cosets can have different generators: Ka = Ka1 can happen where a and a1 may not be equal.

More precisely, let x = Ka = Ka1 and y = Kb = Kb1 be cosets. If we multiply x = Ka and y = Kb using (*) we get xy = Kab, but if we view x and y as x = Ka1 and y = Kb1 we obtain xy = Ka1b1. Clearly, what is needed is:

equation

In this case we say that the multiplication Ka Kb = Kab is well defined. This condition on K is equivalent to K being normal in G.

Lemma. The following conditions are equivalent for a subgroup K of G.

1. K is normal in G.

2. Ka Kb = Kab is a well defined multiplication of right cosets.

Proof. (1) ⇒ (2). If K img G, let Ka = Ka1 and Kb = Kb1, that is img and img We must show that Kab = Ka1b1, that is ab(a1b1)−1 img K. Compute

equation

because aKa−1K. This is what we wanted.

(2) ⇒ (1). If a img G we must show that aka−1 img K for all k img K. Clearly Ka = Ka and Kk = K1, so applying (2) gives Kak = Ka1, that is Kak = Ka. But then (ak)a−1 img K, as required.

Theorem 1. Let K img G and write G/K = {Ka img a img G} for the set of cosets.

1. G/K is a group under the operation Ka Kb = Kab.

2. The mapping img: GG/K given by img(a) = Ka is an onto homomorphism.

3. If G is abelian, then G/K is abelian.

4. If img is cyclic, then G/K is also cyclic; in fact, img

5. If |G: K| is finite then |G/K| = |G: K|; if is finite then img

Proof. (1) The operation on G/K is well defined by the Lemma. The unity of G/K is K = K1 because Ka K1 = Ka = K1 Ka for all Ka in G/K. We have Ka Ka−1 = K1 = Ka−1 Ka, so the inverse of the coset Ka is (Ka)−1 = Ka−1. Finally, associativity in G/K is inherited from G:

equation

(2) We have img(a) img(b) = Ka Kb = Kab = img(ab) for all a, b img G, so img is a homomorphism. It is clearly onto.

(3) If G is abelian, Ka Kb = Kab = Kba = Kb Ka, proving (3).

(4) Let img so every coset in G/K has the form Kak for some integer k. If img is the map in (2), then Kak = img(ak) = img(a)k = (Ka)k by Theorem 1 §2.5. It follows that img as required.

(5) As |G: K| is finite, |G/K| = |G: K| is the definition of the index |G: K|. If is finite, then |G/K| = |G|/|K| is (2) of Lagrange's theorem.

Thus, if n ≥ 2 in img then img as additive groups. If K is a normal subgroup of a group G, write G/K = {Ka img a img G} as in Theorem 1. We make two definitions:

The group G/K of all cosets of K in G is called the factor group of G by K.

The homomorphism img: GG/K where img(a) = Ka is called the coset map.

Hence the unity of G/K is K = K1, and inverses are given by (Ka)−1 = Ka−1. It is important for a student of group theory (and ring theory for that matter) to develop skill in working with factor groups. All the group theoretic techniques we have developed up to now apply to these groups; the only new aspect is that the elements are now cosets.

Example 1. If G is a group then we always have G img G and {1} img G.

If K = G there is only one coset, so G/K = {G} is the group with one element.

If K = {1}, then Ka = {a} for each a img G, so G/K is the set of all singleton

  subsets of G. The operation is {a}{b} = {ab}, so G/KG in this case.

Example 2. Let img where o(a) = 12, and let img Find all the cosets in G/K and write down the Cayley table.

Solution. Note first that K img G because G is abelian. The cosets are

K = {1, a4, a8}, Ka = {a, a5, a9}, Ka2 = {a2, a6, a10}, Ka3 = {a3, a7, a11}.

Two computations are needed to fill in the Cayley table: Ka · Ka3 = K (because a4 img K) and Ka2 · Ka3 = Ka5 = Ka (because a5 img Ka). Then

img

We have img as Ka2 = (Ka)2, Ka3 = (Ka)3, and K = Ka4 = (Ka)4. This confirms Theorem 1(4) in this case.

Example 3. Let K = {ε, (12)(34), (13)(24), (14)(23)}. Show that K img A4, find all the cosets in A4/K, and write down the Cayley table.

Solution. We showed that K img A4 in Example 3 §2.8. The cosets are

equation

The Cayley table is as shown.

img

Here the fact that K(132) = [K(123)]2 shows that img is cyclic. Of course, this also follows from the fact that |G/K| = 3 is prime.

Example 4. Consider the octic group D4 = {1, a, a2, a3, b, ba, ba2, ba3}, where o(a) = 4, o(b) = 2, and aba = b. Show that Z(D4) = {1, a2} and that D4/Z(D4) is isomorphic to the Klein group K4.

Solution. Write Z = Z(D4) for short. We have a(bak) = bak+3bak+1 = (bak)a for each k, so bakZ. Similarly, ab = ba3ba and a3b = baba3 show that aZ and a3Z. Hence Z⊆ {1, a2}. However, a2b = ba2, so a2 commutes with both generators a and b of D4. This implies that a2 img Z, so Z = {1, a2}.

Of course, Z = Z(D4) is normal in D4, and the cosets are

equation

Thus D4/Z = {Z, Za, Zb, Zba}, and the Cayley table is as shown.

img

This is evidently the noncyclic group of order 4, that is D4/ZK4.

Example 5. Let img where o(a) = 18, and let img Find the order of the element Ka5 in G/K.

Solution 1. As K = {1, a6, a12}, we have |G/K| = |G|/|K| = 18/3 = 6. Then, from Lagrange's theorem, the order o(Ka5) of Ka5 is 1, 2, 3, or 6. Now

equation

Hence o(Ka5) is not 1, 2, or 3, so it must be 6.

Solution 2. img because gcd (5, 18) = 1, so img. Hence img by Theorem 1. Because |G/K| = 6, this means that o(Ka5) = 6.

The next theorem provides a useful method of proving that a group is abelian. We include it for reference later.

Theorem 2. Suppose a group G has a subgroup K ⊆ Z(G) such that G/K is cyclic. Then G is abelian.

Proof. Let img If a, b img G, this means that we can write Ka and Kb in the form Ka = Kgm and Kb = Kgn. Thus a = kgm and b = k1gn where k and k1 are in K (and hence are central in G by hypothesis). But then

equation

This shows that G is abelian.

The Derived Subgroup

If H img G there is a useful test for determining when a factor group G/H is abelian. To motivate it, consider the following way of deciding that two cosets Ha and Hb commute:

equation

With this in mind, an element in a group G of the form aba−1b−1 is called a commutator and is denoted

equation

Hence, if H img G then G/H is abelian if and only if H contains every commutator.

The commutator subgroup or derived subgroup G′ of G is defined by

G′ = {all finite products of commutators from G}.

To see that G′ really is a subgroup of G, note that 1 = [a, a] is in G′ and that G′ is clearly closed under the operation of G. The fact that G′ is closed under inverses follows from the first of the following easily verified properties of commutators.

1. [a, b]−1 = [b, a].

2. g[a, b]g−1 = [gag−1, gbg−1] for all g img G.

These facts reveal the relationship between G′ and the abelian factor groups of G.

Theorem 3. Let G be a group and let H be a subgroup of G.

1. G′ is a normal subgroup of G and G/G′ is abelian.

2. G′ ⊆ H if and only if H is normal in G and G/H is abelian.

Proof. We have already established that G′ is a subgroup of G. Since (2) ⇒ (1) by taking H = G′, we prove only (2). If H img G, the above argument shows that G/H is abelian if and only if every commutator belongs to H, that is, if and only if G′ ⊆ H. Hence it remains to show that G′ ⊆ H implies that H img G. If G′ ⊆ H, let g img G and h img H. Then

equation

Thus gHg−1H, so H img G as required.

Hence G′ = {1} if and only if G is abelian. Since G is abelian if and only if Z(G) = G, this contrasts the way G′ and Z(G) measure the commutativity of the group G.

Theorem 3 asserts that G′ is the smallest normal subgroup H of G with the property that the factor group G/H is abelian. This fact can be very useful in computing G′, as Example 6 illustrates.

Example 6. Compute img where D4 = {1, a, a2, a3, b, ba, ba2, ba3}, where o(a) = 4, o(b) = 2, and aba = b.

Solution. In Example 4 we showed that the center of D4 is Z = {1, a2} and that D4/Z is abelian. Hence img by Theorem 3 and so, because |Z| = 2, either img or img But img is impossible because D4/{1} ≅ D4 is not abelian. Hence img

Exercises 2.9

1. In each case find the cosets in G/K, write down the Cayley table of G/K, and describe the group G/K.

(a) G = D6 and K = Z(D6)

(b) G = Q and K = Z(Q)

(c) G = A × B, A and B arbitrary groups, and K = {(a, 1) img a img A}

(d) img where o(a)= 8 and o(b) = 2, and img

2. An integer n is called an exponent for a group G if gn = 1 for every g in G. If n is an exponent for G, show that it is an exponent for every factor group G/K.

3. If img o(a)= 24, let img and img

(a) In G/K, find the order of the elements Ka2, Ka3, Ka4, and Ka5.

(b) In G/H, find the order of the elements Ha2, Ha3, Ha4, and Ha5.

4. Let img where o(a)= 8 and o(b) = 12.

(a) If img find the order of K(a4, b) in G/K.

(b) If img find the order of K(a2, b) in G/K.

5. Let img where o(a) = 6, o(b) = 2, and aba = b.

(a) If img find the order of Ka2, Ka3, Ka5, and Kba in G/K.

(b) If img find the order of Ka2, Ka5, and Kba2 in G/K.

6. If Q denotes the quaternion group, show that Q/Z(Q) has order 4. Is it cyclic or isomorphic to the Klein group? Support your answer.

7. Show that img is an infinite abelian group in which every element has finite order.

8. Let KHG be finite groups, with K img G. Show that H/K = {Kh img h img H} is a subgroup of G/K, and

9. If K img G and o(g)= n, g img G, show that the order of Kg in G/K divides n.

10. If K img G has index m, show that gm img K for all g img G.

11. If K img G has index m and if gcd (m, n) = 1, show that K contains every element of G of order n.

12. Let G be a finite group and let K img G. If G/K has an element of order n, show that G has an element of order n.

13. Let K img G. In each case, if both K and G/K have the given property, show that G also has the property.

(a) Trivial center.

(b) Every element has finite order.

(c) Every element has order a power of a fixed prime p.

(d) Finitely generated.

14. If K img G has prime index p, show that G = KKaimgKap−1 is a disjoint union for some a img G.

15. If img is generated by X, and if K img G, show that G/K is generated by {Kx img x img X}.

16. Let H be a subset of G that is closed under the group operation. If g2 img H for all g img G, show that H is a normal subgroup of G and G/H is abelian.

17. If G is abelian, let T(G) denote the set of elements in G of finite order.

(a) Show that T(G) is a subgroup of G—the torsion subgroup.

(b) Call G a torsion-free group if T(G) = {1}. Show that G/T(G) is torsion free.

(c) Call G a torsion group if T(G) = G. If H is a subgroup of G, show that G is a torsion group if and only if both H and G/H are torsion groups.

18. Let KHG be groups, where K img G and |G: K| is finite. Show that |G/K: H/K| is also finite and that |G/K: H/K| = |G: H|.

19. Find G′ in each case.

img

20. Show that G′ is a characteristic subgroup of G for every group G.

21. Show that (G × H)′ = G′ × H′.

22. If H is a subgroup of G, show that H′ ⊆ HG′. Show that this may not be equality.

23. Let K img G.

(a) If KH where H is a subgroup of G, show that H/K is a subgroup of G/K.

(b) If img is a subgroup of G/K, show that img where H ={h img G img Kh img img is a subgroup of G containing K.

24. If K img G and KG′ = {1}, show that KZ(G) and that img.

25. Let K img G.

(a) Show that [Ka, Kb] = K[a, b] for all a, b img G.

(b) If KG′, show that (G/K)′ = G′/K.

26. Let KZ(G) be a subgroup such that img where xixj = xjxi in G for all i and j. Show that G is abelian. (This extends Theorem 2.)

27. Let KHG be groups with K characteristic in G. If H/K is characteristic in G/K, show that H is characteristic in G. [See Exercise 24 §2.8.]

28.

(a) Show that |G: Z(G)| cannot be a prime for any group G.

(b) Show that G = D4 is a nonabelian group G such that G/Z(G) is abelian.

29. If k|n, k ≥ 2, show that Dn has a normal subgroup K such that Dn/KDk. [Hint: If Dn is generated by a and b where o(a)= n, o(b)= 2, and aba = b, take img]

30. If K = {ε, (12)(34), (13)(24), (14)(23)}, show that S4/KD3. [Hint: Exercise 3 §2.8.]

31. Let G = C4 × C8.

(a) Find subgroups H and K of G such that HK but G/H6 ≅ G/K.

(b) Find subgroups P and Q of G such that G/PG/Q but P6 ≅ Q.

2.10 The Isomorphism Theorem

There is a connection between normal subgroups, homomorphisms and factor groups. The main relationship between these concepts is embodied in the isomorphism theorem, which is the principal result in this section and one of the most useful theorems in group theory. To describe it, we begin by identifying two subgroups associated with a group homomorphism α: GH. The first is

equation

This is a subgroup of H as was shown in Corollary 2 of Theorem 1 §2.5. We now turn to a subgroup of G determined by α: GH:

The kernel of α, defined by ker α = {k img G img α(k) = 1}.

Theorem 1. Let α: GH be a group homomorphism.

1. α(G) is a subgroup of H.

2. ker α is a normal subgroup of G.

Proof. (1) This is Corollary 2 of Theorem 1 §2.5.

(2) We have 1 img ker α because α(1) = 1. If k, kimg ker α, then

equation

Hence kkimg ker α and k−1 img ker α, so ker α is a subgroup. If g img G and k img K then

equation

This shows that g(ker α)g−1 ⊆ ker α for all g img G, and so proves that ker α img G.

Note that the image of a homomorphism α: GH need not be normal in H. For example, if K is any subgroup of H, define the inclusion mapping ı: KH by ı(k) = k for all k img K. This is a one-to-one homomorphism, but ı(K) = K need not be normal in H.

Theorem 1 shows that kernels of homomorphisms from G are normal in G. Conversely, every normal subgroup of a group G arises as the kernel of some homo-morphism with G as domain:

Theorem 2. If K img G, then K = ker img where img: GG/K is the coset mapping.

Proof. The coset map img is defined by img(g) = Kg for all g img G and is a homo-morphism by Theorem 1 §2.9. Because K is the unity of the group G/K, we have g img ker img if and only if Kg = K, if and only if g img K. Hence ker img = K.

Many important subgroups are kernels of naturally occurring homomorphisms; indeed, the easiest way to verify that a subgroup of a group G is normal in G is often to exhibit it as the kernel of a homomorphism with G as domain.

Example 1. The absolute value homomorphism img given by z img |z| has kernel the circle group img

Example 2. The kernel of the determinant homomorphism A img det A from img is the special linear group img

Example 3. If G is a group and g img G has finite order n, let img be the exponent mapping given by α(k) = gk. Then img by Theorem 2 §2.4.

Example 4. Show that An img Sn by exhibiting An as a kernel.

Solution. Define the sign of a permutation σ img Sn by img

Then the sign mapping α: Sn → {1, − 1} given by α(σ) = sgnσ is a homomorphism (see Exercise 29 §1.4). Clearly ker α = An.

Example 5. The trivial homomorphism GH is the only one with G as kernel.

It is clear that a homomorphism α: GH is onto if and only if α(G) = H, that is, if and only if the image α(G) is as large a subgroup of H as possible. The next theorem shows that α is one-to-one if and only if ker α is as small as possible.

Theorem 3. If α: GH is a homomorphism, then α is one-to-one if and only if ker α = {1}.

Proof. If α is one-to-one, let g img ker α. Thus α(g) = 1 = α(1), so g = 1 because α is one-to-one. Hence ker α = {1}. Conversely, let ker α = {1} and suppose that α(a) = α(b) where a and b are in G. Then α(ab−1) = α(a)α(b)−1 = 1, so ab−1 img ker α = {1}. This shows that ab−1 = 1 and hence that a = b. Thus α is one-to-one.

Theorem 3 is used frequently to test when a homomorphism is one-to-one.

We now come to one of the most useful theorems in group theory.

Theorem 4. Isomorphism Theorem35. Let α: GH be a group homomorphism and write K = ker α. Then

equation

Proof. Write K = ker α for simplicity, and define

equation

First img is well defined; that is, Kg = Kg1 implies that α(g) = α(g1). In fact,

equation

Hence img is well defined (⇒) and one-to-one (⇐). As img is clearly onto α(G), it remains to show that it is a homomorphism. But

equation

holds for all Kg and Kg1 in G/K.

If G is a group, a group of the form α(G) where α: GH is some homomorphism is called a homomorphic image of G. Hence the isomorphism theorem shows that the factor groups of G and the homomorphic images of G are the same set of groups up to isomorphism.

Remark. The diagram to the right depicts the mappings α and img in the isomorphism theorem. Here K = ker α as in the theorem, and the mapping img: GG/K is the coset mapping. Note that img is a factorization

img

of the (arbitrary) homomorphism α as a composite where img is onto and img is one-to-one. Indeed, img for all g img G. Moreover, img is the only homomorphism G/KH with the property that img Indeed, if this condition holds then the action of img is determined: img for all Kg in G/K. Hence:

Corollary. Let α: GH be a group homomorphism. Then α factors uniquely as img where img: GG/ker α is the coset map, and img is defined in Theorem 4. Note that img is onto, and img is one-to-one.

The isomorphism theorem is a marvelous result. It sheds light on nearly every situation to which it is applied. It is used as follows: If we want to show that G/KH, we find an onto homomorphism GH with kernel K. As a bonus, the fact that K is a kernel automatically proves that it is normal in G. Examples 6–9 illustrate the use of the isomorphism theorem.

Example 6. If G is a cyclic group, show that img or img

Solution. Let img and define img by α(k) = ak for all img This is an onto homomorphism and ker α = {k img ak = 1}. If o(a) is infinite, ker α = {0} and the isomorphism theorem gives img If o(a) = n, then img and img =img

Example 7. Let K img G and K1 img G1. Show that (K × K1) img (G × G1) and

equation

Solution. We define α: (G × G1) → (G/K) × (G1/K1) by α(g, g1) = (Kg, K1g1). It is routine to verify that this is an onto homomorphism, and ker α = K × K1. The isomorphism theorem now gives all our assertions.

Example 8. Show that img where img is the circle group.

Solution. We define img by α(x) = e2πxi. We have

equation

so α is a homomorphism. It is clearly onto, and

equation

Thus, img and the isomorphism theorem does the rest.

If we are interested in determining all homomorphisms α: GG1, the fact that α(G1) is isomorphic to G/(ker α) is useful because sometimes we can determine the normal subgroups of G. In Example 9 §2.5, we showed that there are at most six homomorphisms: S3C6, and hence at most 6 from D3C6. Using the isomorphism theorem, we can show that in fact there are only two.

Example 9. Write D3 = {1, a, a2, b, ba, ba2}, where o(a) = 3, o(b) = 2, and aba = b, and write img where o(c) = 6. Show that there are only two homomorphisms, D3C6, the trivial one and

equation

Solution. We know that D3 has only three normal subgroups: {1}, D3, and img. Thus if α: D3C6 is a homomorphism, ker α must be one of them. It is impossible that ker α = {1} because then α(D3) ≅ D3 would be a nonabelian subgroup of C6. If ker α = D3 then α is the trivial homomorphism. So assume that img. In this case, let img: D3D3/K be the coset map. If α exists, the corollary to the isomorphism theorem guarantees that α = σimg, whereσ: D3/Kα(D3) is an

img

isomorphism. In this case D3/K = {K, bK} is cyclic of order 2, so α(D3) is the (unique) subgroup of order 2 in C6; that is α(D3) = {1, c3}. Clearly, σ(K) = 1 and σ(bK) = c3, so α = σimg is given by

α(bkam) = σimg(bkam) = σ(bkamK) = σ(bK)k · σ(aK)m = (c3)k · 1m = c3k.

We conclude this section with one more result using the isomorphism theorem. Recall (Example 18 §2.5) that the set innG of all inner automorphisms of a group G is a subgroup of the group autG of all automorphisms of G.

Theorem 5. If G is any group, then G/Z(G) ≅ innG.

Proof. If a img G, recall that the inner automorphism σa: GG is defined by σa(g) = aga−1 for all g img G. Then σaσb = σab for all a, b img G (Example 17 §2.5), and so θ(a) = σa defines a group homomorphism θ: G → autG. Clearly, θ(G) = innG, and

equation

The result now follows from the isomorphism theorem.

Example 10. Show that innS3S3. Show further that innS3 = autS3.

Solution. Z(S3) = {ε} is easily verified, so S3 ≅ innS3 by Theorem 5. Hence so, since innS3 ⊆ autS3, it suffices to show that But S3 = {1, σ, σ2, τ, τσ, τσ2} where o(σ) = 3 and o(τ) = 2. So if θ: S3S3 is an automorphism then o(θ(σ)) = o(σ) = 3, so θ(σ) = σ or σ2. Similarly θ(τ) = τ, τσ or τσ2, so there are at most 2 · 3 = 6 choices for θ because img.

Exercises 2.10

1. Let img and img, subgroups of GLimg Show that K img G by exhibiting K as the kernel of a group homomorphism Gimg

2. Show that the following are equivalent for a group homomorphism α: GG1.

(a) α is trivial (b) ker α = G (c) α(G) = {1}

3. Let H be a subgroup of G with |G: H| = 2, and define α: G → {1, − 1} by img. Show that α is a homomorphism and that ker α = H.

4. If α: GG1 is a group homomorphism and if X is a subgroup of α(G), the preimage of X under α is defined by α−1(X) = {g img G img α(g) img X}. For example α−1({1}) = ker α. [Note: The notation α−1 here is not intended to imply that α is an isomorphism.]

(a) Show that α−1(X) is a subgroup of G, normal if X img α(G).

(b) Show that XY if and only if α−1(X) ⊆ α−1(Y).

(c) Show that α−1(XY) = α−1(X) ∩ α−1(Y).

5. Let ρm: GG be the m-power map: ρm(g) = gm. Assume G is abelian and |G| = n.

(a) Show that ker ρm = {g img gd = 1} where d = gcd (m, n).

(b) If m and n are relatively prime, show that ρm is an automorphism.

(c) If img is cyclic, show that every automorphism of G arises as in (b).

6. Let α: GG1 be a group homomorphism with ker α = K. For a img G, show that Ka = {g img G img α(g) = α(a)}.

7. If α: GG1 is a group homomorphism and both α(G) and ker α are finitely generated, show that G is finitely generated.

8. Find all group homomorphisms

img

9. If K = {ε, (12)(34), (13)(24), (14)(23)}, is there a group homomorphism α: S4A4, with ker α = K? Support your answer.

10. Determine if there exists an onto group homomorphism in each case:

(a) α: S3K4

(b) α: S3C3

(c) α: S3C2

(d) img

11. If G is a group, let θ: GG × G be defined by θ(g) = (g, g).

(a) Show that θ is a one-to-one group homomorphism.

(b) Show that the following conditions are equivalent: (1) G is abelian; (2) θ(G) is normal in G× G; and (3) θ(G) = ker img for some homomorphism img: G × GG.

12. Show that a group G is simple if and only if every nontrivial group homomorphism GG1 is one-to-one.

13. If G is a simple group, show that there is a nontrivial group homomorphism GG1 if and only if G1 has a subgroup isomorphic to G.

14. If n is odd, show that there are at most 36 group homomorphisms DnA4.

15. If |G| ≥ 2 and autG is cyclic, show that G is abelian and that autG is finite and of even order. [Hint: Theorem 2 §2.9 and Theorem 5.]

16. If aut G is simple, show that G is abelian or G/Z(G) is simple. [Hint: Exercise 16 §2.8.]

17. Let α: GG1 be a group homomorphism, as shown in the figure at the right.

(a) Show that img. [Hint: Show α([a, b]) = [α(a), α(b)] for all a, b img G.]

(b) If img: GG/G′ and img are the coset maps, show that a unique homomorphism img exists such that img (see the diagram).

18. If G = H × K and K1 = {(1, k) img k img K}, show that K1 img G, K1K and G/K1H.

19. Let img let K = {A img det A = 1} and let K1 = {A img det A = ± 1}.

(a) Show K img G and img   

(b) Show K1 img G and img

20. Let img If img show that K img G and img

21. Show that img where img is the circle group.

22. Show that img

23. If img define img by τa,b(x) = ax + b for all img It follows that img is a subgroup of img Show that img is a normal subgroup of G and img

24. Consider img as a group under addition. For n ≥ 2, show that img and img—all additive groups.

25. If G is abelian, let K = {(g, g, g) img g img G}. Show that K img G × G × G and G × G × G/KG × G.

26. If G/KH, show that there is an onto homomorphism α: GH with ker α = K.

27. If α: GG1 is a group homomorphism and K img G with ker αK, show that α(K) img α(G) and α(G)/α(K) ≅ G/K.

28. Let G be a finite abelian group. Show that the following conditions are equivalent for an integer m: (1) gm = 1 in G implies that g = 1; and (2) every element g img G has an mth root, that is, g = am for some a img G. Compare your results with those of Exercise 16 §2.6.

29. Let img

(a) Show that G is a subgroup of img and that img

(b) Show that img

30. Use the isomorphism theorem to show that, if m|n, then img

31. Let s = lcm(m, n). Show that img is isomorphic to a subgroup of img [Hint: Think of img as img]

32. Show that every infinite homomorphic image of img is isomorphic to img

33. Describe the homomorphic images of each group.

(a) img   (b) A simple group G    (c) A4 [Hint: Exercise 3 §2.8.]

34. If |G| ≥ 3, show that G has at least two automorphisms. [Hint: Theorem 5.]

35. Let α: GG1 be an onto group homomorphism. If X is a subgroup of G1, define α−1(X) = {g img G img α(g) img X} as in Exercise 4. If X img G1 show that α−1(X) img G and G/α−1(X) ≅ G1/X.

36. If X and Y are additive abelian groups, let hom (X, Y) denote the set of all group homomorphisms α: XY. If α, β img hom (X, Y), define α + β: XY by (α + β)(x) = α(x) + β(x) for all x img X.

(a) Show that hom (X, Y) is an abelian group under this addition.

(b) Show that img for every additive abelian group Y.

(c) Show that img where d = gcd (m, n). [Hint: If e = n/d, define img by img where img and img]

37. If G is a group and gi img G for all i ≥ 0, let [gi) = (g0, g1, g2, img) denote an infinite sequence from G. Define [gi) = [hi) if and only if gi = hi for all i ≥ 0 and define [gi) · [hi) = [gihi). Write Gω = {[gi) img gi img G}.

(a) Show that Gω is a group with the preceding multiplication.

(b) Show that G0 = {[gi) img g0 img G, gi = 1 for all i ≥ 1} is a normal subgroup of Gω, and Gω/G0Gω.

(c) Let F denote the set of mappings img and, if f, g img F, define fg img F by fg(i) = f(i) · g(i) for all img Show that F is a group. What is the relationship between F and Gω? Support your answer.

38. If K img G show that C(K) img G and G/[C(K)] is isomorphic to a subgroup of autK, where C(K) = {a img G img ak = ka for all k img K}. [Hint: Theorem 5.]

2.11 An Application to Binary Linear Codes

The value of mathematics in any science lies more in disciplined analysis and abstract thinking than in particular theories or techniques.

—Alan Tucker

Coding theory is concerned with the transmission of information over a channel that is affected by noise. The noise causes errors, and the general aim is to detect such errors when they occur and to correct them if possible. Such codes are used every day in communication systems such as radio, television, and telephone; in data storage systems such as those used by banks; in the internal circuits of computers; and in many other systems where information is being processed. With the advent of computers, information is often expressed in digital form, that is as strings of 0s and 1s which computers can easily handle. Consequently we deal with binary codes that are based on img

General coding theory originated in the 1940s, primarily with the work of Claude Shannon. He created a mathematical theory of information and proved that certain codes exist which can transmit information at near optimal rates with arbitrarily small chance of error. In 1950, Richard W. Hamming discovered the error-detecting and error-correcting codes that now bear his name. Many of these codes are widely used today.

Example 1 concretely illustrates many of the features of general coding.

Example 1. Suppose that a spacecraft is orbiting the moon, and assume that the message 1 or 0 is to be sent instructing the mission commander to land or not. Because of static interference (noise) the probability36 is 0.1 that an error will occur during transmission (and hence a probability of 0.9 that no error will occur). To ensure accuracy, the earth station transmits five signals: 11111 instead of 1 and 00000 for 0. The spacecraft computer receives a five-digit message and decodes it by a simple majority: It concludes that 11111 was sent if more 1s than 0s are received and that 00000 was sent otherwise. For example, if it receives 11001 it concludes that 11111 was sent. Thus, the spacecraft computer will get the wrong message if and only if three or more errors occur in transmission and (assuming successive errors occur independently) the probability of this happening37 is 0.00856. This probability is less than 1 %, even though there is a 10% chance of error on any one transmission. This decision method is called maximum likelihood decoding.

Example 1 is a good illustration of the way coding works. A sender has a message to send (say, 1 in Example 1). It is encoded (as 11111) and transmitted over a noisy channel where it is received (as, say, 11001) and decoded (as 1) before being sent to the receiver. In Example 1, the coding process can detect errors and correct them with a probability of less than 0.01 of being wrong.

In general, it is desirable to have more messages than 1 or 0 available for encoding and transmission. For convenience (and due to the ubiquity of computers) we assume that our messages, and the encoded messages to be transmitted, are strings of 0s and 1s. We use the following notation. If n ≥ 1, let

equation

denote the direct product of n copies of the (additive abelian) group img The elements of Bn are called words of length n and, for convenience, we write them as strings of 0s and 1s rather than as n-tuples. Thus, 110101 in B6 stands for (1, 1, 0, 1, 0, 1). We call the individual 0s and 1s the bits of the word (an abbreviation for binary digits). A subset C of Bn, with |C| ≥ 2, is called an n -binary code (or simply an n-code). The words in C are called code words.

We describe the general coding process in the diagram.

img

A set of words, called message words, is given in Bk. They are paired with a set C of longer words in Bn, nk, which will actually be transmitted. Thus C is an n-code, and the process of passing from a message to the corresponding code word is called encoding. Only code words are transmitted but, as some bits may be altered during transmission, words other than code words may be received. The sole purpose of the encoding process is to enable the receiver to detect errors and, if there are not too many, to correct them. The encoding and transmission processes are usually quite simple. The message words in Bk are paired with code words in Bn in such a way that passing back and forth is easy. A common method is to add extra bits (called check bits) to the end of the message so that the message itself forms the first k bits of the code word (making retrieval easy). The transmission process is more complex, and the design of codes that are easy and inexpensive to transmit (using, say, shift registers) is an important problem that we do not consider here. The most mathematically interesting part of the process is decoding. A method must be devised to detect bit errors in the received word and, hopefully, to correct them and so reconstruct the transmitted code word. The transmission and decoding part of the process begins and ends with code words, so we concentrate on constructing codes and pay less attention to encoding and retrieving.

In Example 1, the 5-code {00000, 11111} has so few code words that a system (majority rule) of decoding can correct errors with a small probability of error. However, sometimes (for example, when retransmission is easy and inexpensive) all that is needed is to detect errors. Example 2 gives one such system that is commonly used.

Example 2. Parity-check Codes are n-codes that are constructed as follows. The message words are the elements of Bn−1, and we form the code words by adding one extra bit at the end, selecting it so that the total number of 1 s is even (equivalently, the sum of the bits (in img) is 0). Such words are said to have even parity. Thus, the 4-parity-check code C is

img

If a member of C is transmitted and one error occurs, the received word will have an odd number of 1s (odd parity) and so the error is detected. This code can thus detect any odd number of errors, but it cannot detect an even number of errors and it cannot correct any errors. Nonetheless, it is used in banking (the last digit of an account number is often a control digit) and in the internal arithmetic of digital computers.

Nearest Neighbor Decoding

Many important error-correcting codes operate in the following way. A method is found to define the distance between two words in Bn. Then a code CBn is found whose members are so far apart that, if any one bit (say) in a code word c is changed, the new word img is still closer to c than to any other word in the code. Thus, if c is transmitted and one error occurs, the received word img can be corrected by replacing it with the code word closest to it. We state this more compactly as follows.

Nearest Neighbor Decoding Let C be an n -code. If a word img is received, it is decoded as the code word in C closest to it.(If more than one candidate appears, choose arbitrarily38.)

Codes can be constructed that will correct any finite number of errors using nearest neighbor decoding.

Of course the whole thing depends on the existence of an appropriate distance function on Bn. If a word c is transmitted and t errors occur, the received word img will differ from c in exactly t bits. This is the distance between c and img

More precisely, let img and img be words in Bn. The Hamming distance 39 img between img and img is the number of coordinates at which their corresponding bits differ. Thus, if img and img where the img and img are the bits, then img is the number of indices i such that img Define the Hamming weight of img by img Thus, img is the number of 1s occurring as bits of the word img

The following theorem gives some fundamental properties of the Hamming weight and distance functions. The proof uses the fact that Bn is an additive group under componentwise operations. Thus two words are added by adding corresponding bits modulo 2. For example,

equation

Note that the unity is the word 000 img 0, each of whose bits is 0, which we denote 0. Also, img for each word img in Bn, but we write img for clarity.

Theorem 1. Let img and img be words in Bn.

1. img

2. img

3. imgif and only ifimg

4. img

Proof. (1) A bit of img is a 1 if and only if img and img differ at that coordinate. Hence the number of bits of img that are 1 s equals the number of coordinates where img and img differ. This is (1).

(2), (3). We leave the proofs to the reader.

(4) Write img and img so that img Then, using (1), condition (4) becomes wt(x + y) ≤ wtx + wty. Now let xi and yi denote the ith bits of x and y, respectively. Then the wt(x + y) is the number of values of i for which xi + yi = 1. Hence (4) certainly holds if xi + yi = 1 implies that xi = 1 or yi = 1. But this implication is clear because xi = 0 = yi implies that xi + yi = 0.

Properties (2), (3), and (4) of Theorem 1 justify calling d a distance function on Bn. The first two are clearly true of ordinary distance. With respect to

property (4), we may regard img, and img as the vertices of a triangle (see the figure). Then (4) asserts that the length of one side of a triangle is not greater than the sum of the lengths of the other two sides. For this reason we call (4) the triangle inequality.

img

This geometric terminology for Hamming distance is useful for discussing nearest neighbor decoding. If img is a word in Bn and r ≥ 0 is a real number, the set

equation

is called the ball of radius r about w or simply the r-ball about img). We use this to describe how to construct a code C that can detect (or correct) t errors.

Suppose that a code word c is transmitted and a word img is received with s errors, where 1 ≤ st. Then s is the number of coordinates at which the digits of c and img differ; that is, img Hence St(c) consists of all possible received words where at most t errors have occurred. We first assume that C has the property that no code word lies in the t-ball of another code word. Because img and img this means that img is not a code word and that the error has been detected. If we strengthen the assumption on C to require that the t-balls about code words are pairwise disjoint, then img belongs to a unique ball (that about c), so img will be correctly decoded as c.

To describe when this happens, let C be an n-code. The minimum distance d of C is defined to be the smallest distance between two distinct code words in C. That is,

equation

Theorem 2. Let C be an n -code with minimum distance d. Assume that nearest neighbor decoding is used.

1. If t + 1 ≤ d, then C can detect 40 t errors.

2. If 2t + 1 ≤ d, then C can correct t errors.

Proof. (1) If c img C, the t-ball St(c) contains no other code word because t < d. Hence C can detect t errors by the preceding discussion.

(2) If 2t + 1 ≤ d, it suffices (by the preceding discussion) to show that the t-balls about distinct code words are pairwise disjoint. But if cc′ in C and img then the triangle inequality gives

equation

by the hypothesis, a contradiction.

Example 3. The following 7-code has minimum distance 3, so it can detect 2 errors and correct 1 error.

{0000000, 0101010, 1010101, 1110000, 1011010, 0100101, 0001111, 1111111}.

If c is any word in Bn, a word img satisfies img if and only if img and c differ in exactly r bits. Hence there are exactly nr such words img (where nr is the binomial coefficient), because there are nr ways to choose r bits of c to change. Therefore

equation

This leads to a useful bound on the size of error-correcting codes.

Theorem 3. Hamming Bound. If an n-code C can correct t errors, then

equation

Proof. Write N = n0 + n1 + img + nt. The t-balls centered at distinct code words each contain N words, and there are |C| of them. Hence they contain N|C| distinct words (being pairwise disjoint). Hence N|C| ≤ 2n because |Bn| = 2n. This proves the theorem.

An n-code C is called perfect if there is equality in Theorem 3 or, equivalently, if every word in Bn lies in exactly one t-ball about a code word. Such codes exist. For example, if n = 3 and t = 1, then 30 + 31 = 4 and the Hamming bound is 23/4 = 2. The 3 -code C = {000, 111} has minimum distance 3, so by Theorem 2 it can correct 1 error. Hence C is perfect. We present another example of a perfect code later.

Binary Linear Codes and Coset Decoding

Up to this point we have regarded any nonempty subset of Bn as an n-code. However, many important codes are subgroups. The group Bn has order 2n so, by Lagrange's theorem, each subgroup has order 2k for some k = 0, 1, img, n. Given integers k and n, with 1 ≤ kn, an additive subgroup C of Bn of order 2k is called an (n,k)-binary linear code (or simply an (n,k)-code). Note that we do not regard the trivial subgroup (k = 0) as a code.

Example 4. The code {00000, 11111} in Example 1 is a (5, 1)-code.

Example 5. The n-parity-check codes in Example 2 are (n, n − 1)-codes, because the sum of two words of even parity also has even parity.

Example 6. {0000, 0101, 1010, 1111} is a (4, 2) -code. The following is a (4, 3)-code: {0000, 0010, 0101, 0111, 1000, 1010, 1101, 1111}.

Many of the properties of the general n-codes take a simpler form for linear codes. The first part of the next theorem gives a much easier way to find the minimum distance of a linear code, the second and third parts strengthen Theorem 2, and the fourth part reformulates the Hamming bound.

Theorem 4. Let C be an (n, k) -code with minimum distance d.

1. img41

2. C can detect t errors if and only if t + 1 ≤ d.

3. C can correct t errors if and only if 2t + 1 ≤ d.

4. If C can correct t errors, then n0 + n1 + img + nt ≤ 2nk.

Proof. (1) Write img If img then img by the definition of d. Hence d′ ≥ d. However, Theorem 1 gives img for all img in C, so img because img (C is a group). Hence dd′.

(2) Assume that C can detect t errors. If img the t -ball about img contains no other code word (see the discussion preceding Theorem 2). In particular, it does not contain the code word 0, so img Hence t + 1 ≤ d by (1). The converse is part of Theorem 2.

(3) If C corrects t errors, the t-balls about code words are pairwise disjoint (see the discussion preceding Theorem 2). It suffices to show that wtc ≥ 2t + 1 for all c img C, c ≠ 0, since then d ≥ 2t + 1.

So assume, on the contrary, that wtc ≤ 2t. We show that then St(0) ∩ St(c) ≠ ∅, a contradiction. Since cSt(0), we have wt(c) > t, so c has more than t ones as bits. Form img by changing exactly t of these ones to zeros, and leaving the other bits of c as they were. Then img, so img. But c has at most 2t ones as digits (wt(c) ≤ 2t), so img will have at most t ones. Hence img; that is img that is img. So St(0) ∩St(c) ≠ Φ, as required.

(4) Because |C| = 2k, this assertion restates Theorem 3.

In practice, an (n, k)-code C contains a large number of words, so implementing nearest neighbor decoding by computing the distance between a received word and all 2k code words is impractical at best. Fortunately, methods exist for reducing the amount of work required. One of these methods, called coset decoding, is based on the fact that the group Bn is partitioned into cosets by the subgroup C. In fact, there are 2n/2k = 2nk cosets img where img The method depends on the following notion.

In each coset of C in Bn, choose a word e of minimum weight, called the coset leader for that coset. Note that there may be more than one candidate for coset leader. For example, if C is the code in Example 3 and img the coset

equation

has two members of minimum weight 2.

After choosing the coset leaders, we can easily state the decoding procedure.

Coset Decoding Let C be an (n, k) -code. If a word img is received, and if e is any coset leader for img decode img as img

Theorem 5. Coset decoding is nearest neighbor decoding.

Proof. Let C be an (n, k)-code. If a word img is received and e is any coset leader in img then img is a code word in C (because e is in img). We must show that img is as close to c as any other element d of C. We have img so img by the choice of e in C. Hence

equation

which is what we wanted.

Example 7. Consider the (6, 3)-code:

equation

If img and img are received, decode them using coset decoding.

Solution. The cosets generated by img and img are

equation

One of the coset leaders in img is e = 001000, so img decodes as img However, img has three potential coset leaders: f = 010010, g = 001001, and h = 100100. These leaders decode img as 001110, 010101, and 111000, respectively. Note that C has minimum distance 3, so it will correct one error by Theorem 4. Since img is one error away from 100011 (in C), the code corrects img But img for every word c in C, so the code does not correct img Note that 001110, 010101, and 111000 are all the elements of C at distance 2 from img

Given an (n, k)-code C for which |C| = 2k is not too large, we can carry out coset decoding by constructing a table (called a standard array for C), the rows of which are the various cosets img of C in Bn. The coset C = 0 + C is listed in the top row with 0 in column 1. (Note that 0 is the coset a for C.) In general, if e is any coset leader for img then img and we place the elements of this coset in a row of the table with e in column 1 and e + c in the column headed by c for each c img C. We then decode as follows: If we receive a word img we locate it in the table (so img where e is a coset leader) and decode it as the code word c at the head of its column. Here is an example.

Example 8. Construct the standard array for the (4, 2)-code C = {0000, 0110, 1011, 1101}.

img

row, choose any element of B4 not in C, say 1111, and construct the coset

equation

Next, we choose a coset leader, say, e1 = 0100, (0010 would do as well), and obtain row 2 of the table by adding e1 to the elements of row 1 in order. Thus, for example, the word 1111 in column 3 is the sum of e1 and the word 1011 (in C) at the head of column 3.

We complete the rest of the table in the same way. To form any row, we choose an element of B4 not yet listed, find a coset leader in its coset, and list the coset as a row. The remaining coset leaders are e2 = 1000 and e3 = 0001 (each the unique word of minimum weight in its coset).

With the table complete, decoding is easy. For example, if we receive img we decode img as c = 1011 because img is in column 3 of the table.

This method is impractical for large linear codes. For example, a (40, 10) -code has 230 > 109 cosets, so finding the coset leaders is practically impossible. Hence large codes are constructed using more systematic methods.

Matrix Methods

One convenient way to obtain codes is by using matrix multiplication. Here we take the original messages to be the elements of Bk. We regard them as 1 × k row matrices with entries from img and encode by multiplying by a fixed binary matrix (entries from img). We use the usual rules for matrix multiplication, except that we do arithmetic modulo 2.

Example 9. The Hamming (7, 4)-code.42 We use the binary matrix

equation

The message words are the elements of B4; for example, u = 1011 is encoded as uG = 1011001 because of the matrix product

equation

In the chart below, the code words corresponding to all entries of B4 appear on the right. Here each nonzero code word has weight at least 3, so the code can detect two errors and correct one error by Theorem 4.

equation

Observe that the first four columns of the matrix G in Example 9 form the 4 × 4 identity matrix I4. This ensures that the first four digits of each code word uG form the original message word. The general situation is described using the following terminology.

An (n, k)-code C is called a systematic code if each message word in Bk forms the first k digits of exactly one code word. A k × n matrix of the form43

equation

is a standard generator matrix if Ik is the k × k identity matrix and A is a k × (nk) binary matrix. Thus, the matrix G in Example 9 is a 4 × 7 standard generator matrix G = [I4A] where

equation

The code itself is given as C = {uG img u img Bk}.

Theorem 6. Let G be a k × n standard generator matrix. Then

equation

is a systematic (n, k)-code. Conversely, every systematic (n, k)-code is given in this way by a standard generator matrix G.

Proof. Define σ: BkBn by σ(u) = uG for all u img Bk. Then σ is a group homomorphism because matrix multiplication satisfies img As σ is clearly onto C, this shows that C is a subgroup of Bn. In fact, σ is one-to-one. To see this, write G = [IkA], where A is k × (nk). Then

equation

Hence σ is one-to-one because img implies that [uuA] = [vvA], when img Thus Bk and C are isomorphic groups and, in particular, |C| = |Bk| = 2k. Thus C is an (n, k)-code; it is systematic because σ(u) = [uuA] for all u img Bk. This proves the first part of Theorem 6; we leave the converse as Exercise 26.

Example 10. The (6, 3)-code

equation

in Example 7 is systematic, and the reader can verify that it is generated by the standard generator matrix

equation

That is, C = {uG img u img B3}.

If C is a systematic (n, k)-code, we can easily write down a standard generator matrix for C. Because C is systematic, it contains a word ci whose first k digits form row i of Ik. Let G be the k × n matrix whose rows are c1, c2, img, ck in order:

equation

Then G is a standard generator matrix and C = {uG img u img Bk} (See Exercise 26). Incidentally, we say that C is generated by G when C = {uG img u img Bk}. In this case C consists of 0 and all sums of (1 or more) of the generating words c1, c2, img, ck. This is illustrated in Example 11.

Example 11. Both the codes {0000, 0101, 1010, 1111} and

equation

in Example 6 are systematic with matrices img and img.

On the other hand, the (7, 3)-code in Example 3 is not systematic. Nonetheless, every (n, k)-code is close to being systematic in the sense that it contains k words with the following property: If F is the k × n matrix with these words as rows, F contains every column of the k × k identity matrix Ik (Exercise 27).

The use of a standard generator matrix is a convenient method of generating (n, k)-codes not only because retrieval is easy but also because a k × n matrix has only kn entries to store, whereas the code contains 2k words of n entries each. Moreover, the process of encoding with a systematic code is simple: Multiply the message word by the generator matrix. Hence it is not surprising that matrix methods give a simple way to detect and correct errors.

To understand why, let C be a systematic binary (n, k)-code with standard generator matrix G = [IkA], where A is a k × (nk) binary matrix. The parity-check matrix44 for C is the n × (nk) matrix given in block form by

equation

If img is a word in Bn, the word wH in Bnk is called the syndrome of img Note that each of G and H completely determines the other, so either matrix determines the code C.

Example 12. The Hamming (7, 4)-code in Example 9 has the generator matrix

equation

Hence the parity-check matrix is H = img.

In Example 12, the reader can verify that GH = 0 is the zero matrix. This relation holds in general.

Lemma. If G and H are the standard generator matrix and the parity-check matrix of a systematic(n, k)-code, then GH = 0.

Proof. Write G = [IkA] so that img Then block multiplication gives

equation

where A + A = 0 because A is binary and x + x = 0 for all img

Theorem 7. Orthogonality Theorem. Let C be a systematic (n, k)-code with parity-check matrix H.

1. img

2. Words img and img in Bn lie in the same C-coset if and only if wH = vH.

Proof. (1) Let G = [IkA] be the generator matrix for C, so img Define α: BnBnk by img for all img Then α is a group homomorphism because img and (1) amounts to showing that C = ker α. We first verify that α is onto. If img let img be the word whose first k bits are zero and which ends with img Then

equation

Hence α is onto, so imα = Bnk. Now the isomorphism theorem (Theorem 4 §2.10) gives Bn/(ker α) ≅ Bnk, so |Bn|/|ker α| = |Bnk|. Therefore | ker α| = 2k and so | ker α| = |C|. Then to prove that C = ker α, it suffices to show that C ⊆ ker α. But if c img C, then c = uG for some u img Bk (Theorem 6), so α(c) = cH = uGH = u0 = 0 by the lemma. Hence C ⊆ ker α.

(2) For img and img in Bn, we have a chain of equivalences

equation

where the first equivalence comes from Theorem 1 §2.6, the second is by (1), and the third is because img

The orthogonality theorem enables us to reformulate the coset decoding algorithm entirely in terms of the parity-check matrix.

Syndrome Decoding Let C be a systematic (n, k)-code with parity-check matrix H. If img is received, compute its syndrome wH and find a word e img Bn of minimal weight with the same syndrome (that is, wH = eH). Decode img as img

The advantage of this method is that it requires knowing only the syndromes of the coset leaders (rather than the entire standard array), and sometimes the coset leaders can be discovered without finding the whole array.

Nearest neighbor decoding, as we have described it, is complete decoding in the sense that every received word is decoded. However, in many cases (especially where retransmission is easy) a better approach is to use a partial decoding procedure that corrects t errors and calls for retransmission when more than t errors are detected. We conclude by describing one such algorithm.

In this section, we have merely touched the surface of algebraic coding theory. For example, these results generalize with very little change if an (n, k)-code is defined to be a k-dimensional subspace of an n-dimensional vector space V over a finite field F (in our discussion, V = Bn and img). Even more sophisticated coding algorithms exist that use ring theory and field theory as well as group theory and linear algebra (see Section 6.7 for one such application).45

Exercises 2.11

1. Find the Hamming weight of each word.

(a) 10110110 (b) 11010110
(c) 00101011011 (d) 010110101011

2. Find the Hamming distance between each pair of words.

(a) 101101 and 010101 (b) 10110101 and 01110111
(c) 1110111 and 0001000 (d) 10110111 and 01001011

3. Show that img for all img and img in Bn.

4. What is the maximum value of img when img Describe the pairs of words img and img in Bn with img as large as possible.

5. Let img be the word obtained from img by changing every bit.

(a) Show that img for all img

(b) Show that img for all img

6. Let C be the (7, 3)-code in Example 3. Find the nearest neighbors to each of the following words in B7 and so correct them (if possible).

imj

7. How many errors can be detected or corrected by each of the following codes?

(a) img

(b) img

8. Let c be a word in Bn and let 0 ≤ tn. Show that img

9.

(a) Show that the Hamming bound is equality if t = 1 in the (7, 4)-Hamming code.

(b) What is the maximum number of errors that an (8, 3)-code can correct? (c) Is there a (7, 2)-code of minimum distance 5?

10.

(a) If a systematic (n, 2)-code corrects one error, use the Hamming bound to show that n ≥ 5 and find a (5, 2)-code that corrects one error.

(b) If a systematic (n, 2)-code corrects two errors, use the Hamming bound to show that n ≥ 7. Show that no (7, 2)-code can correct two errors. Is there an (8, 2)-code that corrects two errors? Justify your answer.

11.

(a) If an (n, 3)-code corrects two errors, show that n ≥ 9.

(b) Find a (10, 3)-code that corrects two errors. It can be shown that there is no (9, 3)-code that corrects two errors.

12. Given r ≥ 2, write n = 2r − 1 and k = 2rr − 1 so that nk = r. Define H to be the n × r parity-check matrix consisting of all n = 2r − 1 nonzero elements of Br with Ir forming the last r rows. The corresponding (n, k)-code is called a Hamming code. (The (7, 4)-Hamming code is the case r = 3). Show that every Hamming code corrects one error.

13. If a code word c is transmitted and img is received, show that coset decoding will correctly decode img if and only if img is a coset leader in img

14. Suppose that an (n, k)-code C has the property that each word e img Bn, with wtet, is a coset leader in e + C. Show that C corrects t errors by using coset decoding.

15.

(a) Show that no (4, 2)-code can correct single errors.

(b) Construct a (5, 2)-code that can correct a single error.

16.

(a) Show that no (6, 3)-code can correct two errors.

(b) Construct a (6, 3)-code that can correct a single error. (c) Show that no (7, 3)-code can correct two errors.

17. Given words img, define their product vw to be the word whose ith digit is the product img where img and img are the ith digits of img and img

(a) Show that img

(b) Deduce the triangle inequality: img (See Theorem 1.)

(c) Show that equality holds in (b) if and only if the ith bit of img is 0 whenever the ith bit of img is 1.

18. If img show that img with equality if and only if the ith bit of img is 1 whenever the ith bit of img is 1. [Hint: Preceding exercise.]

19. If C is an (n, k)-code, img and img show that img is an (n, k + 1)-code.

20. Write down the standard generator matrix G and the parity-check matrix H for each of the following systematic codes.

(a) C = {00000, 11111}.

(b) C = any systematic (n, 1)-code.

(c) The code in Exercise 7(a). (d) The code in Exercise 7(b).

21. List the codes generated by each standard generator matrix.

(a) img (b) img
(c) img (d) img

22. If C is the (n, n − 1)-parity-check code (Example 2), show that C is systematic and describe the standard generating matrix G and the parity-check matrix H.

23. (Requires matrix algebra) Prove Theorem 7(a) without using the isomorphism theorem by writing each img such that wH = 0 as img where u consists of the first k bits of img and img is the last nk bits of img

24. (Requires matrix algebra) Let C and C′ be (n, k) -codes, with standard generator matrices G and G′, and parity-check matrices H and H′, respectively.

(a) Show that C = C′ if and only if G = G′.

(b) Show that C = C′ if and only if H = H′.

25. Let C be an (n, k)-code.

(a) Show that either each word in C has even weight or exactly half have even weight.

(b) Show that either each word in C has nth bit 0 or exactly half have nth bit 0.

(c) Generalize.

26. Show that every systematic (n, k)-code C is generated by a k × n standard generator matrix G; that is, C = {uG img u img Bk}. [Hint: Let c1, c2, img, ck be the rows of Ik; that is, ci has the ith bit 1 and all other bits 0. If img is the unique element of C with ci as its first k bits, take G = [IkA], where the rows of A are img]

27. (Requires linear algebra) Show that every (n, k)-code C contains k words c1, c2, img, ck such that the k × n matrix K = img contains every column of the k × k identity matrix Ik. [Hint: Regard C as a vector space over img and let {b1, img, bk} be a basis. If B = img, carry B to reduced row-echelon form BR and let ci be row i of R.]

Notes

18. The term “identity” is often used here but it has other meanings in algebra. So we use the term unity.

19. A set with an associative binary operation, but possibly no unity, is called a semigroup.

20. This proof will not be used below and so may be omitted at a first reading. By contrast, the theorem will be used hundreds of times.

21. See Nicholson, W.K., Linear Algebra with Applications, 7th ed., McGraw-Hill Ryerson, 2012 (Theorem 2 §3.2).

22. The name honors the Norwegian mathematician Niels Henrik Abel.

23. If X and Y are sets the set difference is defined by img.

24. The name honors Felix Klein. This group is also called the four group.

25. To avoid confusion, we sometimes denote the unity of a group G by 1G when other groups are present.

26. The notation Z(G) comes from zentrum, the German word for center.

27. Homomorphisms were first used explicitly (for permutation groups) by Jordan in 1870.

28. The term isomorphism comes from isos, meaning equal, and morphe, meaning shape.

29. See, for instance, Clay, J.R., The punctured plane is isomorphic to the unit circle, Journal of Number Theory, 1, (1964), 500–501.

30. Recall (Section 0.4) that a partition of a nonempty set X is a collection of nonempty subsets of X (called the cells of the partition) which are pairwise disjoint (distinct cells are disjoint) and every element of X is in some cell (hence in exactly one cell).

31. In fact img where img is the inner automorphism determined by g.

32. Thompson, J. G., and Feit, W., Solvability of Groups of Odd Order, Pacific Journal of Mathematics, 13 (1963), 775–1029.

33. John Thompson was awarded the Fields Medal in 1970, the highest honor a mathematician can receive.

34. More information can be found in Chapter 17 of “Finite Groups” by D. Gorenstein, Chelsea, 1980.

35. This result goes back to Camille Jordan (1838–1922) in his book Traité des Substitutions (1870), where the concept of a homomorphism was introduced.

36. We treat probability informally here. The probability that an event occurs is the long-term proportion of the time that the event does indeed occur. Thus probabilities are numbers between 0 and 1. A probability of 0 means that the event in question is impossible; a probability of 1 means that the event is certain to occur; and a probability of 0.5 means that the event is as likely as not to occur.

37. The probability is computed as img. It is based on the assumption that at most one error occurs in each digit transmitted and that these errors occur independently.

38. If it is feasible, retransmission may be called for in this case.

39. The name honors Richard W. Hamming. Distance functions are also called metrics.

40. If C can detect (correct) t or fewer errors, we say simply that C detects (corrects) t errors.

41. Because of this the minimum distance of a linear code is sometimes called the minimum weight of the code.

42. This code was the first nontrivial example of an error correcting code given in the groundbreaking paper in which information theory was originated (Shannon, C.E., A mathematical theory of communication, Bell Systems Technical Journal 27 (1948), 623–656).

43. If A and B are k × m and k × n matrices, the notation [A B] indicates the k × (m + n) matrix with A occupying the first m columns and B occupying the last n columns. The matrix [A B] is said to be given in block form.

44. Systematic binary codes are often defined using the parity-check matrix. Then the transpose of H is referred to as the parity-check matrix.

45. An introduction to the subject is given in Pless, V., Introduction to the Theory of Error Correcting Codes, New York: Wiley, 1982. A more thorough treatment (with an extensive bibliography) is that by MacWilliams, F.I., and Sloan, N.J.A., The Theory of Error Correcting Codes, Vols. I and II, New York: North Holland, 1977. Finally, a useful survey is contained in Chapter 4 of Lidl, R., and Pilz, G., Applied Abstract Algebra, New York: Springer-Verlag, 1984.

..................Content has been hidden....................

You can't read the all page of ebook, please click here login for view all page.
Reset
18.222.37.169